Download as pdf or txt
Download as pdf or txt
You are on page 1of 54

IONIC EQUILIBRIUM Page # 3

IONIC EQUILIBRIUM

An ionic Equilibrium exists between the unionised electroyte molecules and the ions that result from
ionisation
A + B A+ + B–

 Types of keq


Self ionization Acid dissociation Base dissociation Salt Sparingly
of water constant constant hydrolysis soluble salt
H2O H++OH + –
H A H+ +A– BOH B++ OH– – +
A B +H2O HA+BOH AgCl Ag++Cl–

 Concept of electrolyte

Strong electrolyte Weak electrolyte
A  B–  A   B– AB A+ + B–
a 0 0 a 0 0
0 a a a (1 – ) a a
 100% disssociation ( = 1) <1
 no equilibrium equilibrium
 it is irreversible it is reversible process.

 Self ionization of water

1000 / 18
Concentration of H2O ([H2O]) =  55.5 M
1
Water is very weak electrolyte
H2O H+ + OH–
55.5
55.5 – 10–7 10–7 10–7 (at 25oC)

[H ] [OH– ]
 k eq 
[H2 O]
Constant
 Kw = keq [H2O] = [H+] [OH–]

 Kw = [H+] [OH–], at 25oC, [H+] = [OH–] = 10–7


o –14
Kw (25 C) = 10

Self ionization constant of water
let, Kw1 at temperature T1 , Kw2 at temperature T2

Kw 2 H  1 1 
 log   –  T   Kw 
K w1 2.303 R  T1 T2  ,

394 - Rajeev Gandhi Nagar Kota, Ph. No. 0744-2209671, 93141-87482, 93527-21564
IVRS No. 0744-2439051, 0744-2439052, 0744-2439053 www.motioniitjee.com, email-info@motioniitjee.com
Page # 4 IONIC EQUILIBRIUM

 For calculation of  :
+ –
H2O H + OH
c 0 0
c (1 – ) cc
Kw = c × c
kw
   c = 5 5.56 M    1.8  10 –2 Kw
c

pH (pH scale is given by Sorension)

 potent Hydrogen ion



strength
at 25°C
+
 pH = – log [H ] pH scale
p OH = – log [OH–]
p Kw = – log Kw Neutral
at any temperature :
[H+] = [OH–] O Acidic basic
[H+] [OH–] = Kw 7 14

 – log [H+] – log [OH–] = – log [Kw]

 pH + pOH = p Kw

at 25oC at 80oC
Kw = 10–14 Kw = 10–12
pH + pOH = 14 pH + pOH = 12
25oC 90oC
pH + pOH = 14 pH + pOH = 12

Some concept regarding pH calculation


 Concept 1 :  Concept 2:

5L 1L 3L 2L 5L
pH = 3  pH = ? pH = 3 + pH = 3  pH = ?

 Concept 3:  Concept 4:

2L 8L 10 L 2L 8L 10 L
pH = 3 + H2O = pH = ? pH = 13 + water = pH = ?

Ca lcula te pH (i) Ca lcula te pOH


 Note : 7
pH (ii) pH = 14- pOH

394 - Rajeev Gandhi Nagar Kota, Ph. No. 0744-2209671, 93141-87482, 93527-21564
IVRS No. 0744-2439051, 0744-2439052, 0744-2439053 www.motioniitjee.com, email-info@motioniitjee.com
IONIC EQUILIBRIUM Page # 5

 Concept 5:

pH = 3 + pH = 4 = pH = ?

2 : 1

 Note : The final pH of solution after mixing two solution is in between the previous solution pH.

 Concept 6:  Concept 7:

10 M HCl
–8
V V 2V
pH = 3 + pH = 11  pH = ? pH = ?

 Note :
When [H+] > 10–5  [H+]water neglected
When [H+] < 10–5  [H+]water considered
H2O H+ + OH–
55.56 10–8
55.56 – x 10–8 + x x
 x (10–8 + x) = 10–14  x = .94 × 10–7
 [H+] = 10–8 + .94 × 10–7 = 1.04 × 10–7
 pH = 7 – log (1.04)

 Concept 8:

2
10 M HCl
pH = ?

 Dissociation of weak acid :

H+ A– H+ + A–
c
c (1– ) c c
c 2
Ka  for weak acid < < 1 Ka = c2
1– 


Kw
[H+] = c = c Ka , pH = – log c Ka , for bases [H+] = c  Kb
 Note :
(a) HA1 c Ka 1
||

[H  ]1 Ka 1
HA2 c Ka 2 
 
[H ] 2 Ka 2

394 - Rajeev Gandhi Nagar Kota, Ph. No. 0744-2209671, 93141-87482, 93527-21564
IVRS No. 0744-2439051, 0744-2439052, 0744-2439053 www.motioniitjee.com, email-info@motioniitjee.com
Page # 6 IONIC EQUILIBRIUM

 So HA2 is stronger acid than HA1


(b) HA1 c1 Ka 1

[H ]1 c1 ka1
HA2 c2 Ka 2 
[H ]2 c 2 Ka2

 The acidic strength of two weak acid can be compared by Ka value only in the case when their
concentration are same. Otherwise the procedure of case (b) should be followed .

pH of mixture of two acids

 Case (I) (Strong acid + weak acid)

Question : HCl (0.1 M) + CH3COOH (0.2 M)



Ka = 10–5
CH3COOH CH3COO– + H+
t=0 0.2 0 0. 1
t = teq 0.2 – x x 0.1 + x (x is very small)
  
0.2 x 0.1
x  0.1
 pH = – log (0.1) = 1  [CH3COO–] = ?  10 – 5   [CH3 COO– ]  2  10 – 5
0.2

 Case (II-A) (weak acid + weak acid ) (kas are incomparable)

Question : HA1 (0.1 M, Ka 1 = 10–5)

HA2 (0.2 M, Ka 2 = 10–9)

HA1 H+ + A1 – HA2 H+ + A2–


0.1 0 0 0.2 0 0
0.1 – x x+y x 0.2 – y x+y y ( Ka 1 >> Ka 2

       x > > y)
0.1 x x 0.2 x y
x and y are also very less
x. x
10 –5  [H+] = x = 10–3 ,  pH = 3 [A1– ] = [A2–] =
10 –1
 Case (II-B) (weak acid + weak acid ) (kas are comparable)

394 - Rajeev Gandhi Nagar Kota, Ph. No. 0744-2209671, 93141-87482, 93527-21564
IVRS No. 0744-2439051, 0744-2439052, 0744-2439053 www.motioniitjee.com, email-info@motioniitjee.com
IONIC EQUILIBRIUM Page # 7

Question : HA 1
(0.1) Ka 1 = 10–5

HA2 (0.2) Ka 2 = 10–6


HA1 H+ + A1 – HA2 H+ + A2–
0.1 0 0 0.2 0 0
0.1 – x x+y x 0.2 – y x+y y
0.1 x+y x 0.2 x+y y

x (x  y)
 now K a1 
0.1
x (x  y) (x  y) y
 Ka 1  , Ka 2 
c1 c2

 [H  ]  x  y  c 1 Ka1  c 2 Ka 2

 weak polyprotic acids or bases


(H2S, H3PO4, NH2 – NH2, H2CO3)

(a) H2S H+ + HS– Ka 1 HS– H+ + S2– Ka 2


 
(b) NH2 – NH2 + H+ NH2 – NH3+ Kb 1 NH2 – NH3+ + H+ NH3 – NH3 Kb 2
Cases

 Case I : H2S (0.1 M Ka 1 = 10–4 , Ka 2 = 10–9)

 H2S H+ + H S– H S– H+ + S2–
0.1 0 0 x x 0
0.1 – x x+y x–y x–y x+y y
     
0.1 x x x x y
Ka 1 > > Ka 2 x >> y

x2 x y
10 – 4   x 10 –5  10 –9
0.1 x
[H+] = x = [HS–] = x = [S2–] = y =

 Case II : HCl H+ + Cl–


0.2 0 0
0 0.2 0.2
 H2S H+ + HS– H S– H+ + S2–
0.1 0.2 0 x 0.2 + x 0
0.1 – x 0.2 + x + y x x–y 0.2 + x + y y
     
0.1 0.2 x x 0.2 y
[H+] = 0.2  pH = – log (0.2)

394 - Rajeev Gandhi Nagar Kota, Ph. No. 0744-2209671, 93141-87482, 93527-21564
IVRS No. 0744-2439051, 0744-2439052, 0744-2439053 www.motioniitjee.com, email-info@motioniitjee.com
Page # 8 IONIC EQUILIBRIUM

 Note : Strong acid controls the pH.


 Now , [H+] = 0.2
 Case III : H3PO4 (0.1 M) Ka 1 = 10–5 , Ka 2 = 10–8 , Ka 3 = 10–11

 H3PO4 H+ + H2PO4– H2PO4– H+ + HPO42–


0.1 0 0 x x
0.1 – x x+y+z x–y x–y x+y y
     
0.1 x x x x y
 HPO42– H+ + PO43–
y y+x
y–z x+y+z z
  
x2
y x z   10 – 5  x  10 – 3  pH = 3
0. 1

 Buffer
 Any solution which resist the change in pH is called Buffer solution.
 Requirement of Buffer
(1) Must have acidic and basic components so that it can consume H+, OH– added to the buffer.
(2) Acidic and basic components should not react with each other.
CH3COOH CH3COO– + H+
acid conjugate base
 
acidic component basic component

do not react with each other.


(3) Buffers can be prepared by following two ways
 Buffer

CH3COOH / CH3COONa NH4OH / NH4Cl


W. A / Canjugate Base W. B / Conjugate acid
 acidic buffer  basic buffer
 0 < pH < 14  0 < pH < 14
 Acidic buffer Basic buffer
CH3COOH / CH3COONa NH4OH / NH4Cl
Ka c1 c2 Kb c1 c2
CH3COOH CH3COO– + H+ NH4OH NH4+ + OH–
c1 c2 0 c1 c2 0
c1 – x c2 + x x c1 – x c2 + x x
     
c1 c2 x c1 c2 x
in presence of CH3COO– dissociation
c2 .x
of CH3COOH is supressed. Kb 
c1

394 - Rajeev Gandhi Nagar Kota, Ph. No. 0744-2209671, 93141-87482, 93527-21564
IVRS No. 0744-2439051, 0744-2439052, 0744-2439053 www.motioniitjee.com, email-info@motioniitjee.com
IONIC EQUILIBRIUM Page # 9

c1
So x is very small x  Kb
c2
c2 x [salt]
Ka  pOH = pKb + log
c1 [base ]
Ka c 1 c2 [salt]
 x= pH  pK a  log pH  pK a  log
c2 c1 [acid]

 Buffer action
 Acidic Basic
CH3COOH / CH3COONa NH4OH / NH4Cl
c1 c2 c1 c2

c2 [c 2 ]
pH = pKa + log c pOH = pKb + log [c ]
1 1

 (a) NaOH is added (x M) (a) HCl is added (x M)


– –
CH3COOH + OH CH3COO + H2O NH4OH + H+ NH4+ + H2O
c1 x c2 c1 x c2
c1 – x 0 c2 + x c1 – x 0 c2 + x
c2  x c2  x
pH  pKa  log pOH  pKb  log
c1 – x c1 – x
 (b) HCl is added (x M) (b) NaOH is added (x M)

CH3COOH CH3COO + HCl NH4OH NH4+ + OH–
c1 c2 x c1 c2 x
c1 + x c2 – x 0 c1 + x c2 – x 0
c2 – x c2 – x
pH  pKa  log pOH  pKb  log
c1  x c1  x
 How to prepare buffer

Acidic buffer Basic buffer


 (a) CH3COOH / CH3COONa (a) NH4OH / NH4Cl
c1 c2 c1 c2
c2 c2
pH  pKa  log pOH  pKb  log
c1 c1
 (b) CH3COOH + NaOH CH3COONa + H2O (b) NH4OH + HCl NH4Cl + H2O
a x a x 0
a–x 0 x a–x 0 x
x x
pH  pKa  log pOH  pKb  log
a–x a–x
 (c) CH3COONa + HCl CH3COOH + NaCl (b) NH4Cl + NaOH NH4OH + NaCl
a x 0 a x
a–x 0 x a–x 0 x
a–x a–x
pH  pKa  log pOH  pKb  log
x x

394 - Rajeev Gandhi Nagar Kota, Ph. No. 0744-2209671, 93141-87482, 93527-21564
IVRS No. 0744-2439051, 0744-2439052, 0744-2439053 www.motioniitjee.com, email-info@motioniitjee.com
Page # 10 IONIC EQUILIBRIUM

 Parameters of buffer

 Buffer range Requirement of good buffer


The range of pH a buffer can work nicely (1) pH = pKa
is called buffer range resist both OH– and H+ in equal amount
salt c2
pH = pKa + log (2) pH = pKa + log c
acid 1

 [salt] 
after buffer action max. ratio   = 10
 [acid ] 
c2 – x
pH = pKa + log c  x  pH = pKa + 1
1

 [salt] 
min. ratio   = 0.1 both will be approximately same when
 [acid ] 
pH = pKa – 1 c1 and c2 are very large
c1 , c2 > > x
c2
pKa – 1 pKa pKa + 1  pH  pK a  log
c1
 Buffer capacity
number of mol of H added/L of solution
 Buffer Capacity 
Change in pH of buffer
 Buffer of polyprotic acid
H2CO3 / NaHCO3
c1 c2
H2CO3 HCO 3– + H+ Ka 1 HCO 3– H+ + CO 23– Ka 2
c1 c2 0 c2 + x x 0
c1 – x c2 + x – y x+y c2 + x – y y+x y
     
c1 c2 x c2 x y
c2 y
pH  pK a1  log pH  pK a 2  log
c1 c2
 Salt Hydrolysis
Acid + Base  Salt + H2O neutralization
Salt + H2O  Acid + Base  reverse of neutralization  Salt hydrolysis
since salt hydrolysis is an endothermic reaction hence on increasing the temperature, the extent
of hydrolysis increases.
 Types of salt

Case I Case II Case III Case IV


W. A + S. B S. A. + W. B. W. A + W. B. S. A. + S. B.
e.g. CH3COONa e.g. NH4Cl e.g. CH3COONH4 e.g. NaCl

394 - Rajeev Gandhi Nagar Kota, Ph. No. 0744-2209671, 93141-87482, 93527-21564
IVRS No. 0744-2439051, 0744-2439052, 0744-2439053 www.motioniitjee.com, email-info@motioniitjee.com
IONIC EQUILIBRIUM Page # 11

Case : (I) Hydrolysis of WA + SB :


+
CH3COONa  CH3COO– + Na  spectator ion

 
Very strong Very weak
Conjugate base Conjugate acid

H – OH

CH3COOH NaOH

CH3COO– + H+ Na+ + OH–


CH3COONa CH3COO– + Na+

CH3COO + H2O CH3COOH + OH–
__________________________________________
CH3COONa + H2O CH3COOH + Na+ + OH–
__________________________________________________________________________

Note : (1) Only weaker part of salt undergoes hydrolysis. (2) This solution becomes basic.
Case : (II) S.A. + W.B. (NH4Cl) :
NH4+ + H2O NH4OH + H+, This solution becomes acidic
Case : (III) W.A. + W.B. (CH3COONH4) :

CH3COO + H2O CH3COOH + OH–
a x
a–x
NH4+ + H2O NH4OH + H+
a y
a–y
Can be acidic, basic or neutral
 Ka  Kb  acidic Ka  Kb  neutral Ka  Kb  basic
Case : (IV) S. A + S. B (NaCl) :
 No hydrolysis  Solution will be neutral
 Relationship between Ka, Kb, and Kh
Case (I) : W. A. + S.B. (CH3COONa)
CH3COO– + H2O CH3COOH + OH–
[CH3 COOH] [OH– ] [CH3 COOH] [OH– ]  [H ] wK
 Keq  –  Kh  Keq [H2 O]  –   Kh  K
[H2O] [CH3 COO ] [CH3 COO ]  [H ] a

Case (II) : S. A. + W. B. (NH4OH)


+
NH 4
+ H2O NH4OH + H+
[NH4 OH][H ] [NH4 OH][H ]  [OH– ] wK
 Kh    Kh   –  Kh  K
[NH4 ] [NH4 ]  [OH ] b

Case (III) W. A. + W. B. (CH3COO NH4)


CH3COO– + H2O CH3COOH + OH–
+
NH4 + H2O NH4OH + H+
_______________________________________________________
CH3COO– + NH4+ + 2 H2O NH4OH + CH3COOH + H+ + O H–
_______________________________________________________
[NH4 OH] [CH3 COOH] [NH4 OH] [CH3 COOH] [OH– ] [H ] w K
 Kh  –
[CH3 COO ] [NH4 ]   Kh   –   Kh  K . K
[NH4 ] [OH ] –
[CH3 COO ] [H ] a b

394 - Rajeev Gandhi Nagar Kota, Ph. No. 0744-2209671, 93141-87482, 93527-21564
IVRS No. 0744-2439051, 0744-2439052, 0744-2439053 www.motioniitjee.com, email-info@motioniitjee.com
Page # 12 IONIC EQUILIBRIUM

 pH calculation
Case (I) : W. A. + S. B. (CH3COONa  c, Ka) , h  degree of hydrolysis.
– –
CH3COO + H2O CH3COOH + OH
c 0 0
c (1 – h) ch ch

(ch) 2 ch 2 Kh
Kh   h<<1 Kh = ch2 h

c (1– h) 1 – h
  c

Kw 1 1
 pOH  [pKw – pKa – log c]  pH  [pKw  pKa  log c]

 [OH ]  c  h  c. Kh  c.
Ka 2 2

Case (II) :S. A. + W. B. (NH Cl 4  c, Kb)


+
NH 4
+ H2O NH4OH + H+
c 0 0
c (1 – h) ch ch

ch 2 Kh
 Kh  h<<1  Kh = ch2  h 
1– h c

Kw 1 1 kw
 [H ]  c.h  c  Kh  c   pH  [pKw – pKb – log c]  pOH = p  pk b  log c
 
Kb 2 2

Case (III) W. A. + W. B. (CH3COONH4  Ka, Kb , c)


– +
CH3 COO + NH + H2O 4
NH4OH + CH3COOH
c c
c (1 – h) c (1 – h) ch ch

c 2h 2 h2
 Kh  2 2
  K h = h2 1– h  1  h  Kh
c (1– h) (1 – h)2

Kw
 h - - - - - (1)
Ka Kb

[H ] [CH3 COO – ]  [CH3 COOH]


 CH3COOH CH3COO– + H+  Ka  [CH3 COOH]  [H ]  Ka [CH COO – ]
3

ch Kw
  Ka  (h < < 1)  [H+] = Ka . h  [H ]  Ka . (from 1)
c (1– h) Ka . Kb

Ka 1
 [H ]  Kw .  pH  [pKw  pKa – pKb ]
Kb 2
 If , Ka > Kb  pKa < pKb  acidic, Ka = Kb  neutral, Ka < Kb  basic
 Summary of hydrolysis
Kw 1
1. W. A. + S. B. Kh  K pH 
2
[pK w  pK a  log c]
a

Kw 1
2. W. B. + S. A. Kh  K pH 
2
[pK w – pKb – log c]
b

Kw 1
3. W. A. + W. B. K h  K . K pH 
2
[pK w  pK a – pK b ]
a b

394 - Rajeev Gandhi Nagar Kota, Ph. No. 0744-2209671, 93141-87482, 93527-21564
IVRS No. 0744-2439051, 0744-2439052, 0744-2439053 www.motioniitjee.com, email-info@motioniitjee.com
IONIC EQUILIBRIUM Page # 13

 Hydrolysis of salt of polyprotic acid / base (Na2 CO3)


Kw
 CO 23– + H2O HCO3– + OH– Kh1 (CO3 – – ) 
Ka 2
a
a–x x–y x+y
 
x x
– Kw
 HCO3– + H2O H2CO3 + OH– Kh2 (HCO3 )  K
a1

x–y y x+y
  
x y x
Ka1   Ka 2  Kh1   Kh2  Mainly hydrolysis is governed by CO 23– .

– c  Kw c  Kw Kw x y Kw
[HCO3– ]  [OH– ]  [H2CO 3 ] 
  [OH ]  c  Kh1 
Ka 2 ,   Ka 1 ,   Ka 
x  Ka1
1

 Solubility of sparingly soluble salt

Solubility
AgCl (s)   AgCl (aq.) Ag  (aq)  Cl– (aq)
 In ionic equilibrium all the components of equilibrium should be in same phase.
 In case of solubility equilibrium of sparingly soluble salt equilibrium is a heterogeneous equilibrium.
Example (1) : Solubility of AgCl
AgCl (s) Ag+(aq) + Cl–(aq)
[Ag  ] [Cl – ]
Keq 
[AgCl]
 Ksp = Keq . [AgCl] = [Ag+] [Cl–]
 It is a endothermic reaction on increasing temperature T   solubility 
Ksp2 H 1 1
 log   – 
Ksp1 2.303 R  T1 T2 

Example (2) : Solubility of Ag2 CO3 Ag2 CO3(s) 2 Ag+ + CO 23–

 Ksp  [Ag ]2 [CO 32 – ] ,  In general , if salt is Mx Ny type, Mx Ny x My+ + y Nx–

 Ksp  [M y  ]x [Nx– ]y
let , Q = [My+]x [Nx–]y Q = ionic product
If , Q = Ksp  Equilibrium (Saturated solution) Q > Ksp  PPt (Super saturated)
Q < Ksp  Unsaturated solution.
 Various case in solubility
 Case (1) : Ag2SO4 (S = 10–3 mol/L)
Ag2SO4 2 Ag+ + SO 24 –
S 0 0
0 2S S Ksp = [Ag+]2 [SO4- - ] = (2 S)2 (S) = 4 S3
= 4 × (10–3)3

394 - Rajeev Gandhi Nagar Kota, Ph. No. 0744-2209671, 93141-87482, 93527-21564
IVRS No. 0744-2439051, 0744-2439052, 0744-2439053 www.motioniitjee.com, email-info@motioniitjee.com
Page # 14 IONIC EQUILIBRIUM

 Case (2) : S (moL / L)  S (gm / L)


10–3
SAg2SO4 (gm/L) = 10–3 × (108 × 2 + 96)
 Case (3) : Ksp given , Solubility = ?
Ag2 SO4 2 Ag +
+ SO24–
S 0 0
0 2S S

S  3 Ksp
 4 S3 = Ksp  mol/L
4
 Case (4) : 70% of the dissolved salt ionizes .
Ag2SO4 2Ag+ + SO24–
S 0 0
S – 0.7S 2 × 0.7S 0.7S
S  3 K sp
 Ksp = (1.4S)2 (0.7S) 
0.98
 Case (5) : Solubility of Ag2SO4 in 0.1 M AgNO3
AgNO3  Ag+ + NO3–
0.1 0 0
0 0.1 0.1
Ag2SO4 (solid) Ag2 SO4 (aq) 2 Ag+ + SO4–
S 0.1
0 0.1 + 2S S
K sp
 KSp = (0.1 + 2S)2 (S)  Ksp  (0.1)2  S  S 
0.01
 Case (6) : Solubility of Ag2SO4 in 0.1M Na2 SO4
Na2SO4 2 Na+ + SO24–
0.1 0 0
0 0.2 0.1
Ag2SO4 (S) Ag2 SO4 (aq) 2 Ag+ (aq) + SO24– (aq.)
S 0 0.1
0 2S 0.1 + S
K sp
 KSp = (2 S)2 (S + 0.1)   4 S2  0.1  S
0.4
 Case (7) : 100 ml 0.01 M AgNO3 + 400 ml 0.02 M Na2SO4, Ksp (Ag2 SO4) = 4 × 10–9
(a) Will any ppt or not ?
 8 
Q = [Ag+]2 [SO24– ] Q = (1/500)2   = 6.4 × 10–8 Q > Ksp  ppt
 500 
(b) After ppt [Ag+] = ? ; [SO42–] = ?
at equilibrium
4S3 = 4 × 10–9 S = 10–3
Number of moles of (Ag2SO4) = 500 × 10–3 = 0.5 m mole
2 Ag+ + SO24– Ag2SO4
1 8
1 – 2x 8–x x
2
 y   7.5  –9
     Ksp  4  10
y 7.5 0.5  500   500  y =?

394 - Rajeev Gandhi Nagar Kota, Ph. No. 0744-2209671, 93141-87482, 93527-21564
IVRS No. 0744-2439051, 0744-2439052, 0744-2439053 www.motioniitjee.com, email-info@motioniitjee.com
IONIC EQUILIBRIUM Page # 15

Solved Example
Ex.1 At 25°C the degree of ionization of water was found pH = – log (4.24 × 10–3) = 3 – log 424 = 3 – 0.6273 = 2.37
to be 1.8 × 10–9. Calculate the ionization constant and ionic Now pH on dilution = 2 × 2.37 = 4.74
product of water at this temperature
[H+] = 1.8 × 10–5 = c
Sol. If x is the degree of ionization of water, then
c 2 c   1.8  10 –5  
H 2O H+ + OH– Ka = = = = 1.8 × 10–5
1–  1–  1– 
c(1 –  ) c c

1000 1   = 0.5
c  [H2 O]   55.56 M 1– 
18
[H+] = ac
[H ][OH ] ( c ) 2
K eq    c 2 (since  is very much less [H ] 1.8  10 –5
[H2O] c(1 –  ) c   3.6  10 – 5 M
 0 .5
than 1)
Let the volume to which 1  of 1 M acetic acid is diluted be V,
Keq = 55.56 × (1.8 × 10–9)2 = 1.8 × 10–16 M then
Kw = [H+] [OH–] = (c)2 = (55.56 × 1.8 × 10–9)2 1 × 1 = 3.6 × 10–5 × V
Kw = 1.0 × 10–14 M2
1 1 10 5
V= –5 = = 0.277 × 104 litres.
3.6  10 3 .6
Ex.2 (a) Calculate the pH of a 0.01 M solution of benzoic
acid, the Ka being 7.3 × 10–5. You can notice that  increases on dilution and it become
(b) 0.2 M solution of Ba(OH)2 is found to be 90% ionised at considerable and cannot be ignored with respect to 1.
25°C. Find the pH of the solution at that temperature.
Sol. (a) First calculate  using the expression,  = Ex.4 A weak base, BOH is titrated with a strong acid HA.
When 10 ml of HA is added, the pH of the solution is 10.2
Ka 7.3  10 –5 and when 25 ml is added, the pH of the solution is 9.1.
  0.085
c 0.01 Calculate the volume of acid that would be required to reach
equivalence point.
pH = ½(pKa – logc) = ½ (4.13 – 0.01) = 2.06
Sol. Let the molarity of HA be ‘M2’ and the molarity and volume
(b) [OH–] = 0.2 × 0.9 × 2 = 0.36 of weak base (BOH) are M1 and V1 respectively.
pOH = 0.44; pH = 13.56 In first case,
BOH + HA BA+ H2O
Ex.3 What is the pH of a 1.0M solution of acetic acid ? mmole before reaction M1V1 10 M2 0 0
To what volume must 1 litre of the solution be diluted so
that the pH of the resulting solution will be twice the (M1V1 – 10M2) 0 10 M2 10 M2
original value ? Given : Ka = 1.8 × 10–5. Since the solution (after addition on 10 ml of HA) contains
weak base (BOH) and salt of its conjugate base (BA), the
Ka
Sol. Let us first calculate  using  = solution behaves like a basic buffer.
c
[Salt ] 10M2
1.8  10 –5  pOH = pKb + log 14 – 10.2 = pKb + log (M V – 10M )
  4.24  10 – 3 [Base] 1 1 2
1
Dividing numerator and denominator of log tem by M2 gives
Since,  is very much smaller than 0.1, so the assumption is
valid
10M2
 [H+] =  = 2.19 × 10–5 M2
3.8  pK b  log
  M1V1 10M2 
+ –5
 [H ] = K a  c  1.8  10  1 = 4.24 × 10 –3  – 
 M2 M2 

394 - Rajeev Gandhi Nagar Kota, Ph. No. 0744-2209671, 93141-87482, 93527-21564
IVRS No. 0744-2439051, 0744-2439052, 0744-2439053 www.motioniitjee.com, email-hr.motioniitjee@gmail.com
Page # 16 IONIC EQUILIBRIUM

Ex.6 Calculate the concentrations of all the species present


10
3.8  pK b  log ...(i) in 0.1 M H3PO4 solution.
v 2 – 10
Given : K1 = 7.5 ×10–3, K2 = 6.2 × 10–8 and K3 = 3.6 × 10–13.
where V2 is the volume of acid required to get equivalence Sol.
point (At equivalence point, M1V1 = M2V2)
In second case, I Step H3PO 4 H  H2PO 4 ; K 1  7.5  10 3
BOH + HA BA+ H2O
II Step H2PO 4 H  HPO42 ; K 2  6.2  10 8
mmole before reaction M1V1 25M2 0 0
mmole after reaction (M1V1 – 25M2) 0 25M2 25M2 III Step HPO 42 H  PO 43 ; K 3  3.6  10 13

25M2 For I Step H3PO 4 H  H2PO 4


14 – 9.1 = 4.9 = pKb + log (M V – 25M )
1 1 2
0.1 M 0 0
Dividing numerator and denominator of log term by M2 (0.1 – c) M c c

25
4.9  pK b  log ...(ii) [H ] [H2PO 4 ] c2 c2
V2 – 25 K1   7.5  10 3 
[H3PO 4 ] (0.1  c ) ( 0 .1  c )
Substracting equation (i) from (ii)
Solving the quadratic equation,
 25   10  c = 0.024
4.9 – 3.8  1.1   log  –  log 
 V2 – 25   V2 – 10 
 [H ]  0.024 M  [H2PO 4 ]  0.024 M

5 V2 – 50 [H3PO 4 ]  0.024 M [H3PO 4 ]  0.1  0.024  0.076 M


Taking antilog, 12.58  V2 = 28.72 ml
2V2 – 50 
The value of K1 is much larger than K2 and K3. Also dissociation
of II and III steps occurs in presence of H+ famished in I step
Ex.5 20 ml of 0.2 M NaOH is added to 50 ml of 0.2 M and thus, dissociation of II and III steps is further surppressed
acetic acid. What is the pH of the resulting solution? due to common ion effect.
Calculate the additional volume of 0.2 M NaOH required
For II Step : H2PO 4 H  HPO 42
making the solution of pH 4.74. The ionization constant of
acetic acid is 1.8 × 10–5. 0.024 0.024 0
Sol. 20 ml of 0.2 M NaOH would react with 20 ml of 0.2 M (0.024 – y) (0.024 + y) y
acetic acid.
The dissociation of H2PO 4 occurs in presence of [H+] furnished
30  0.2 6 in step I.
[Acid] = 30 ml of 0.2 M present in 70 ml =  mole
70 70
[H  ] [HPO 42 ] 8 (0.024  y ) y
20  0.2 6 Thus, K2  or 6.2  10  (0.024  y )
[Salt] = 20 ml of 0.2 M present in 70 ml =  mole [H2PO 4 ]
70 70
y is small  0.024 – y  0.024 and neglecting y2.
pKa = – log 1.8 × 10–5 = 4.74 

0.024 y
[salt] 4 70  6.2   8   y  6.2  10 8
pH =pKa + log = 4.74 + log × 0.024
[ Acid] 70 6

pH = 4.74 + log 0.66 = 4.74 – 0.18 = 4.56 or [HPO 42 ]  K 2  6.2  10 8 M


To make a solution of pH = 4.74, [Acid] = [Salt]
For III Step : HPO 42 H  PO 43
So 25 ml of 0.2 M NaOH must be added to 50 ml of 0.2 M
acetic acid. (6.2×10–9 -x) (0.024 + x) x
Addition volume of NaOH to be added = 25 – 20 = 5 ml

394 - Rajeev Gandhi Nagar Kota, Ph. No. 0744-2209671, 93141-87482, 93527-21564
IVRS No. 0744-2439051, 0744-2439052, 0744-2439053 www.motioniitjee.com, email-hr.motioniitjee@gmail.com
IONIC EQUILIBRIUM Page # 17

Ex.9 Calcium lactate is a salt of a weak organic acid and


[H ] [PO 43 ] (0.024  x ). x is represented as Ca(Lac)2. A saturated solution of Ca(Lac)2
K3  
[HPO 42 ] ( 6.2  10 8  x ) contains 0.13 mol of this salt in 0.5 litre solution. The pOH
of this solution is 5.6. Assuming complete dissociation of
Again neglecting x2 and assuming, 6.2  10 8  x  6.2  10 8 the salt, calculate Ka for latic acid.
Sol. Ca(Lac)2(s)  Ca(Lac)2(aq)  Ca2+ (aq) + 2Lac–(aq)
13 0.024 x
 3.6  10 
6.2  10 8 Lac–(aq) + H2O LaCH (aq) + OH– (aq)

Since it is salt of strong base and weak acid, its pH is calculated


3.6  10 13  6.2  10 8 as
 [PO 34 ] x  9.3  10 19 M.
0.024
1
pH  [pK w  pK a  log c ] pH = 14 – pOH = 14 – 5.6 = 8.4
2
Ex.7 The pH of blood stream is maintained by a proper
balance of H2CO3 and NaHCO3 concentrations. What volume 1
8.4 = [14 + pKa + log 0.52] ( c = 2 × 0.13/0.5)
of 5 M NaHCO3 solution, should be mixed with 10 ml 2
sample of blood, which is 2 M in H2CO3 in order to maintain
pKa = (2 × 8.4) – 14 – log 0.52 = 3.08
a pH of 7.4 Ka for H2CO3 in blood is 7.8 × 10–7 ?
 Ka = 8.31 × 10–4
Sol. Let the volume of 5 M NaHCO3 solution added be x ml.
Number of millimoles of NaHCO3 = 5x
Ex.10 A certain acid - base indicator is red in acid and
Number of millimoles of H2CO3 = 10 × 2 = 20
blue in basic solution. At pH = 5, 75% of the indicator is
[NaHCO 3 ] present in the solution in its blue form. Calculate
For the acidic buffer, pH = pKa + log [H CO ] dissociation constant (Ka) for the indicator and pH range
2 3
over which the indicator changes from 90% red-10% blue
to 90% blue-10% red.
5x
7.4   log(7.8  10 7 )  log Sol. Since Ka is asked, the indicator must be an acid. Let
20
the acid be represented by HIn.
On solving, x = 78.37 ml
HIn H+ + In–
 Volume of NaHCO3 solution required = 78.37 ml.
In acid solution, the indicator will be predominatly present in
Ex.8 An indicator is a weak acid and the pH range of its
the form of HIn (due to common ion effect.) Since in acid solutin
colour is 3.1 to 4.5. If the neutral point of the indictor lies
the the colour is red, this would be due to HIn. In basic solution,
in the center of the hydrogen ion concentrations
the indicator will be predominatly in the form of In–. Since the
corresponding to the given pH range, calculate the
indicator is blue in basic solution, so In– must be blue in colour.
ionization constant of the indicator.
At pH = 5, the indicator is 75% blue. This also means it is
Sol. The hydrogen ion concentrations of the given pH range are
25% red.
For pH 3.1, pH = – log [H+] = 3.1
10 –5  0.75
[H+] = 7.94 × 10–4 M For pH 4.5,  Ka   3  10 – 5
0.25
pH = 4.5 = – log [H+]  [H+] = 3.16 × 10–5 M
pH when it is 90% red & 10% boue
The average of these two hydrogen ion concentrations is
K a  [HIn] 3  10 –5  0.9
–4
(7.9  10 )  (3.16  10 –5
)
+
: [H ] =   2.7  10 – 4
= 4.128 × 10–4 M [In – ] 0.1
2
 pH = 3.56
At this concentration of H+, we will get neutral point of the
indicator, at which [In–] = [HIn] pH when it is 90% blue and 10% red :

 pH = pKHin
K a  [HIn] 3  10 –5  0.1
or [H+] = KHin = 4.128 × 10–4 [H+] =   3.3  10 – 6

[In ] 0.9

394 - Rajeev Gandhi Nagar Kota, Ph. No. 0744-2209671, 93141-87482, 93527-21564
IVRS No. 0744-2439051, 0744-2439052, 0744-2439053 www.motioniitjee.com, email-hr.motioniitjee@gmail.com
Page # 18 IONIC EQUILIBRIUM

Ex.11 Determine the number of mole of Agl which


s 2  2  10 3
may be dissolved in 1.0 litre of 1.0 M CN– solution.
Given Ksp for Agl and Kc for Ag(CN)2 are 1.2 × 10–17 s  2  10 9  4.472  10 5 mole /  .
M2 and 7.1 × 1019 M–2 respectively.
Let the solubility of CaC 2 O 4 be s in the presence of
Sol. Given, Agl( s) Ag  (aq)  I (aq) 0.1 M ammonium oxalate solution.

CaC 2 O 4 Ca 2  C2O 24


; K sp  [ Ag  ] [I ]  1.2  10 17 ...(i)
(NH4 )2 C2O 4  2 HN4  C2O 24
  
Ag (aq)  2 CN (aq) [ Ag (CH2 )] (aq) 0.1 M

Since CaC 2 O 4 is sparingly soluble, the concentration


[ Ag (CH)2 ] 19
; Kf   7.1 10 ...(ii) of C 2 O 24 derived from CaC 2 O 4 in negligible in comparison
[ Ag  ] [CN ] 2
to 0.1.
Let ‘x’ mole of Agl be dissolved in CN– solution, then
K sp  [Ca 2 ] [C 2O 24 ]  s  0.1  2  10 9

Now Agl(s)  2CN [ Ag(CN)2 ]  I
2  10 9
Mole before reaction s   2  10 8 mole / 
0 .1
1 0 0 In the presence a ammonium oxalate, the solubility of
Mole at equilibrium (1 – 2x) x x CaC 2 O 4 drops from 4.472  10 5 to 2  10 4 mole /  .
By equations (i) and (ii). K eq  K sp  K f This effect is called common-ion effect.

[ Ag(CN)2 ] [I ]
K eq   1.2  10 17  7.1 1019 Ex.13 At 25°C, will a precipitate of Mg(OH)2 form
[CN ] 2 in a 1 × 10–4M solution of Mg(NO3)2 if pH of the
solution is adjusted to 9.0. [Ksp (Mg(OH))2 = 8.9
K eq  8.52  10 2 ...(iii) × 10 –12 M 3]. At which minimum pH will the
precipitation start ?

x2 x Sol. If pH = 9.0, [H+] = 1 × 10–9 M, then


 K eq  8.52  10 2  or  29.2
(1  2x )2 1  2x
Kw 1 10 –14
[OH+] =   1 10 – 5 M
 x = 0.49 mole. [H ] 1 10 – 9

The ionic product of Mg(OH)2 in the solution would be


Ex.12 The solubility product of calcium oxalate
[Mg2+] [OH–]2 = (1 × 10–4) (1 × 10–5)2 = 1 × 10–14
is 2 × 10–9 at 25ºC. Calculate the solubility of 3
M
calcium oxalate at this temperature. What will be
its solubility in a 0.1 M solution of ammonium Since, the value of ionic product in smaller than
oxalate ? Assume that ammonium oxalate is
KSP(8.9 × 10–12), so no precipitate of Mg(OH)2 will be
completely ionized in solution. Name the effect,
formed.
which operates on addition of ammonium oxalate
to calcium oxalate. The minimum concentration of OH – needed to
precipitate Mg2+ from the solution is
Sol. The solubility product, K SP of calcium oxalate is
K SP 8.9  10 –12
given by [OH – ]    2.98  10 – 4 M
[Mg 2 ] 1 10 – 4
K SP  [Ca 2 ] [C 2O 24 ]  2  10 9
 Maximum pOH = 3.54
Let ‘s’ be the solubility of calcium oxalate in mole litre–1.
and minimum pH = 14 – 3.54 = 10.46

394 - Rajeev Gandhi Nagar Kota, Ph. No. 0744-2209671, 93141-87482, 93527-21564
IVRS No. 0744-2439051, 0744-2439052, 0744-2439053 www.motioniitjee.com, email-hr.motioniitjee@gmail.com
IONIC EQUILIBRIUM Page # 19

Class Room Problems


Ex.1 Nicotinic acid (Ka = 1.4 × 10–5) is represented Ex.12 Two buffers (X) and (Y) of pH 4.0 and 6.0
by the formula HNiC. Calculate its per cent dissociation respectively are prepared from acid HA and the salt
in a solution which contains 0.10 moles of nicotinic NaA. Both the buffers are 0.50 M in HA. What would
acid per 2.0 litre of solution. be the pH of the solution obtained by mixing equal
volumes of two buffers ? (K HA  1.0  10 5 )
Ex.2 Calculate the conc. of fluoroacetic acid when
[H+] = 1.50 × 10–3 M. Ka of acid = 2.6 × 10–3.
Ex.13 A certain buffer solution contains equal
concentration of X– and HX. Kb for X– is 10–10. Calculate
Ex.3 A saturated solution of o-nitrophenol, has a pH pH of buffer.
equal to 4.53. What is the solubility of o-nitrophenol
in water pKa for o-nitro-phenol is 7.23.
Ex.14 A weak acid HA after treatment with 12 mL
of 0.1 M strong base has a pH of 5. At the end point,
Ex.4 A solution contains 0.1 M H2S and 0.3 M HCl. the volume of same base required is 26.6 mL. Calculate
Calculate the conc. of S2– and HS– ions in solution. Ka of acid.
Given K a1 and K a2 for H2S are 10–7 and 1.3 × 10–13
Ex.15 The [Ag+] ion in a saturated solution of Ag2CrO4 at 25ºC
respectively.
is 1.5 × 10–4 M. Determine Ksp of Ag2CrO4 at 25ºC.

Ex.5 K1 and K2 for dissociation of H2A is 4 × 10–3 and Ex.16 Will a precipitate of Mg(OH)2 be formed in a 0.001 M
1 × 10–5. Calculate A 2 concentration in 0.1 M H2A solution of Mg(NO3)2, if the pH of solution is adjusted to 9? Ksp
solution. Also report [H+] and pH. of Mg(OH)2 = 8.9 × 10–12.

Ex.17 Calculate pH of the following mixture. Given that Ka =


Ex.6 Calculate the pH of a solution which contains
1.8 × 10–5 and Kb = 1.8 × 10–5.
9.9 mL of 1 M HCl and 100 mL of 0.1 M NaOH.
(a) 50 mL of 0.1 M NaOH + 50 mL of 0.05 M CH3COOH.
(b) 50 mL of 0.05 M NaOH + 50 mL of 0.10 M CH3COOH.
Ex.7 What will be the resultant pH when 200 mL of an (c) 50 mL of 0.1 M NaOH + 50 mL of 0.10 M CH3COOH.
aqueous solution of HCl (pH = 2.0) is mixed with 300
(d) 50 mL of 0.1 M NH4OH + 50 mL of 0.05 M HCl.
mL of an aqueous solution of NaOH (pH = 12.0) ?
(e) 50 mL of 0.05 M NH4OH + 50 mL of 0.1 M HCl.
(f) 50 mL of 0.10 M NH4OH + 50 mL of 0.1 M HCl.
Ex.8 A solution contains 0.09 M HCl, 0.09 M CHCl2COOH
(g) 50 mL of 0.05 M NH4OH + 50 mL of 0.05 M CH3COOH.
and 0.1 M CH3COOH. The pH of this solution is 1. If Ka
for acetic acid is 10–5, calculate Ka for CHCl2COOH.
Ex.18 Assuming no change in volume, calculate the minimum
+ – – mass of NaCl necessary to dissolve 0.010 mole AgCl in 100
Ex.9 What are [H ], [A ] and [B ] in a solution that is
litre solution. Kf for [AgCl2]– = 3 × 105 and Ksp of AgCl = 1.0 ×
0.03 M HA and 0.1 M HB ? Ka for HA and HB are
10–10
1.38 × 10–4 and 1.05 × 10–10 respectively. Also report
pH of solution.
Ex.19 Calculate the concentration of Fe 2+ in a solution
containing 0.2 M [Fe(CN)6]4– and 0.10 M CN–. Kf for [Fe(CN)6]4–
Ex.10 Calculate the pH of a solution of given
= 1 × 1024
mixtures. (a) (2g CH3COOH + 3g CH3COONa) in 100 mL
of mixture ; Ka = 1.8 × 10–5. (b) 5 mL of 0.1 M NH4OH
+ 250 mL of 0.1 M NH4Cl; Kb = 1.8 × 10–5 (c) (0.25
mole of acid + 0.35 mole of salt) in 500 mL mixture; Ka
= 3.6 × 10–4.

Ex.11 20 mL of 0.2 M NaOH is added to 50 mL of 0.2


M acetic acid. (Ka = 1.8 × 10–5) (a) What is pH of
solution ? (b) Calculate volume of 0.2 M NaOH required
to make the pH of solution 4.74.

394 - Rajeev Gandhi Nagar Kota, Ph. No. 0744-2209671, 93141-87482, 93527-21564
IVRS No. 0744-2439051, 0744-2439052, 0744-2439053 www.motioniitjee.com, email-hr.motioniitjee@gmail.com
Page # 20 IONIC EQUILIBRIUM

Exercise - I (Only one option is correct)


1. The conjugate acid of NH2– is
(A) NH3 (B) NH2OH (C) NH4+ (D) N2H4 N
10. What is the percentage hydrolysis of NaCN in
80
2. Out of the following, amphiprotic species are solution when the dissociation constant for HCN is
I : HPO32– II OH– III H2PO4– IV HCO3– 1.3 × 10–9 and K w = 1.0 × 10 –14
(A) I, III, IV (B) I and III (C) III and IV (D) All (A) 2.48 (B) 5.26 (C) 8.2 (D) 9.6

11. The compound whose 0.1 M solution is basic is


3. pH of an aqueous solution of NaCl at 85°C should (A) Ammonium acetate (B) Ammonium chloride
be (C) Ammonium sulphate (D) Sodium acetate
(A) 7 (B) > 7 (C) < 7 (D) 0

12. Which of the following solution will have pH close


4. 1 c.c. of 0.1N HCl is added to 99 CC solution of to 1.0 ?
NaCl. The pH of the resulting solution will be
(A) 100 ml of M/100 HCl + 100 ml of M/10 NaOH
(A) 7 (B) 3 (C) 4 (D) 1
(B) 55 ml of M/10 HCl + 45 ml of M/10 NaOH
(C) 10 ml of M/10 HCl + 90 ml of M/10 NaOH
M
5. 10 ml of H2 SO 4 is mixed with 40 ml of (D) 75 ml of M/5 HCl + 25 ml of M/5 NaOH
200
M
H2 SO 4 . The pH of the resulting solution is 13. The  pH of the neutralisation point of 0.1 N
200 ammonium hydroxide with 0.1 N HCl is
(A) 1 (B) 2 (C) 2.3 (D) none of these (A) 1 (B) 6 (C) 7 (D) 9

6. If pKb for fluoride ion at 25°C is 10.83, the ionisation 14. If equilibrium constant of
constant of hydrofluoric acid in water at this
temperature is : CH3COOH + H2O CH3COO– + H3O+
(A) 1.74 × 10–5 (B) 3.52 × 10–3 Is 1.8 × 10–5, equilibrium constant for
(C) 6.75 × 10–4 (D) 5.38 × 10–2 CH3COOH + OH– CH3COO– + H2O is
(A) 1.8 × 10–9 (B) 1.8 × 109
7. The pH of an aqueous solution of 1.0 M solution of
(C) 5.55 × 10–9 (D) 5.55 × 1010
a weak monoprotic acid which is 1% ionised is:
(A) 1 (B) 2 (C) 3 (D) 11
15. If 40 ml of 0.2 M KOH is added to 160 ml of 0.1 M
HCOOH [Ka = 2 × 10–4]. The pOH of the resulting
8. If K1 & K2 be first and second ionisation constant of solution is
H3PO4 and K1 >> K2 which is incorrect.
(A) 3.4 (B) 3.7 (C) 7 (D) 10.3
(A) [H ]  [H2PO–4 ] (B) [H ]  K1[H3PO4 ]
16. A solution with pH 2.0 is more acidic than the
(C) K2  [HPO2–
4 ]
(D) [H ]  3[PO3–
4 ] one with pH 6.0 by a factor of :
(A) 3 (B) 4
9. The degree of hydrolysis of a salt of weak acid and (C) 3000 (D) 10,000
weak base in it's 0.1 M solution is found to be 50%. If
the molarity of the solution is 0.2 M, the percentage 17. The first and second dissociation constants of
hydrolysis of the salt should be an acid H2A are 1.0 × 10–5 and 5.0 × 10–10 respectively.
(A) 100% (B) 50% The overall dissociation constant of the acid will be:
(C) 25% (D) none of these (A) 5.0×10–5 (B) 5.0×1015
–15
(C) 5.0×10 (D) 0.2×105

394 - Rajeev Gandhi Nagar Kota, Ph. No. 0744-2209671, 93141-87482, 93527-21564
IVRS No. 0744-2439051, 0744-2439052, 0744-2439053 www.motioniitjee.com, email-hr.motioniitjee@gmail.com
IONIC EQUILIBRIUM Page # 21

26. 1 M NaCl and 1 M HCl are present in an aqueous


18. An aqueous solution contains 0.01 M RNH 2 solution. The solution is
(Kb= 2 × 10–6) & 10–4 M NaOH. (A) not a buffer solution and with pH < 7
The concentration of OH– is nearly: (B) not a buffer solution with pH > 7
(A) 2.414 ×10–4 M (B)10–4 M (C) a buffer solution with pH < 7
(C) 1.414×10–4 M (D) 2×10–4 M (D) a buffer solution with pH > 7

19. What volume of 0.2 M NH4Cl solution should be 27. the pKa of a weak acid (HA) is 4.5. The pOH of
added to 100 ml of 0.1 M NH4OH solution to produce a an aqueous buffered solution of HA in which 50% of
buffer solution of pH = 8.7 ? the acid is ionized is:
Given : pKb of NH4OH = 4.7 ; log 2 = 0.3 (A) 4.5 (B) 2.5 (C) 9.5 (D) 7.0
(A) 50 ml (B) 100 ml
(C) 200 ml (D) none of these 28. The precipitate of CaF2 (Ksp = 1.7 × 10–10) is
obtained when equal volumes of the following are mixed
20. The pKa of a weak acid, HA, is 4.80. The pKb of a (A) 10–4 M Ca3+ + 10–4 M F–
weak base, BOH, is 4.78. The pH of an aqueous solution (B) 10–2 M Ca2+ + 10–3 M F–
of the corresponding salt, BA, will be: (C) 10–5 M Ca2+ + 10–3 M F–
(A) 8.58 (B) 4.79 (D) 10–3 M Ca2+ + 10–5 M F–
(C) 7.01 (D) 9.22
29. pH of saturated solution of silver salt of monobasic
21. The range of most suitable indicator which should acid HA is found to be 9.
be used for titration of X– Na+ (0.1 M, 10 ml) with 0.1 Find the Ksp of sparingly soluble salt Ag A(s).
–6 Given : Ka(HA) = 10–10
M HCl should be (Given : k b( X – )  10 )
(A) 1.1 × 10–11 (B) 1.1 × 10–10
(A) 2 – 3 (B) 3 – 5 (C) 6 – 8 (D) 8 – 10 (C) 10 –12
(D) None of these

22. How many gm of solid NaOH must be added to


30. When equal volumes of the following solutions
100 ml of a buffer solution which is 0.1 M each w.r.t.
are mixed, precipitation of AgCl (Ksp = 1.8 × 10–10) will
Acid HA and salt Na+ A– to make the pH of solution
occur only with:
5.5. Given pKa(HA) = 5 (Use antilog (0.5)= 3.16)
(A) 10–4 M (Ag+) and 10–4 M (Cl–)
(A) 2.08 × 10–1 (B) 3.05 × 10–3
(C) 2.01 × 10 –2
(D) None of these (B) 10–5 M (Ag+) and 10–5 M (Cl–)
(C) 10–6 M (Ag+) and 10–6 M (Cl–)
23. The solubility of A2X3 is y mol dm–3. Its solubility (D) 10–10 M (Ag+) and 10–10 M (Cl–)
product is
(A) 6y2 (B) 64 y4 (C) 36 y5 (D) 108 y5 31. 50 litre of a solution containing 10–5 mole of Ag+
is mixed with 50 litre of a 2×10–7 M HBr solution. [Ag+]
24. If Ksp for HgSO4 is 6.4 × 10–5, then solubility of in resultant solution is: [Given : Ksp(AgBr)= 5×10–13]
this substance in mole per m3 is (A) 10–5M (B) 10–6M
(A) 8 × 10–3 (B) 6.4 × 10–5 –7
(C) 10 M (D) None of these
(C) 8 × 10–6 (D) None of these

32. The solubility of metal sulphides in saturated


25. Which of the following in most soluble in water ?
solution of H2S {H2S = 0.1 M} can be represented by
(A) MnS (Ksp = 8 × 10–37)
(B) ZnS(Ksp = 7 × 10–16) [M2  ][H2 S]
MS + 2H+ M2+ + H2S ; K eq 
(C) Bi2S3(Ksp = 1 × 10–72) [H ] 2
(D) Ag3(PO4)(Ksp = 1.8 × 10–18) The value of Keq is given for few metal suphide. If
conc. of each metal ion in solution is 0.01 M, which

394 - Rajeev Gandhi Nagar Kota, Ph. No. 0744-2209671, 93141-87482, 93527-21564
IVRS No. 0744-2439051, 0744-2439052, 0744-2439053 www.motioniitjee.com, email-hr.motioniitjee@gmail.com
Page # 22 IONIC EQUILIBRIUM

metal sulphides are selectively ppt at total [H+] = 1 Statement-2 : Solubility product constant of Sr(OH)
2
M in saturated H2S solution. is not affected by dilution.
Metal sulphides MnS ZnS CoS PbS (A) Statement (1) is correct and statement (2) is
correct and statement (2) is correct explanation for
[M2 ][H2S] (1)
K eq  3 × 1010 3 × 10–2 3 3 × 10–7
[H ]2
(B) Statement (1) is correct and statement (2) is
correct and statement (2) is NOT correct explanation
(A) MnS, ZnS, CoS (B) PbS, ZnS, CoS for (1)
(C) PbS, ZnS (D) PbS (C) Statement (1) is true but (2) is false
(D) Statement (1) is false but (2) is true

33. Solid Ba(NO3)2 is gradually dissolved in a 1.0×10–4 M


Comprehension
Na2CO3 solution. At what concentration of Ba2+ will a
Paragraph for Quesiton 38 to 40
precipitate begin to form ?
1.2 of a monprotic acid HA, is titrated with 0.222 M
(Ksp for BaCO3 = 5.1 × 10–9)
NaOH solution. The pH of the solution is monitrored
(A) 4.1 × 10–5 M (B) 5.1 × 10–5 M with pH meter. A portion of the titration curve is shown
–8
(C) 8.1 × 10 M (D) 8.1 × 10–7 M in the diagram.
Expanded titration
curve of
34. Ksp of MX4 and solubility of MX4 is S mol/litre is 12.00 HA vs NaOH
related by: 11.00
1/5 1/4 10.00
(A) S = [Ksp/256] (B) S = [128 Ksp]
9.00
(C) S = [256 Ksp]1/5 (A) S = [Ksp/128]1/4
8.00
7.00
ASSERTION - REASON 6.00
–7
35. Statement-1 : pH of 10 M NaOH solution is 5.00
exist between 7 to 7.3 at 25ºC. 15 16 17 18 19 20
Statement-2 : Due to common ion effect ionization volume of NaOH(mL)
of water is reduced. 38. How many mL of NaOH is required to bring about
(A) Statement (1) is correct and statement (2) is the titration to its equivalence point?
correct and statement (2) is correct explanation for (1) (A) 4.00 (B) 9.00
(B) Statement (1) is correct and statement (2) is (C) 19.00 (D) None of these
correct and statement (2) is NOT correct explanation
for (1) 39. What is the pH of solution at the equivalence
(C) Statement (1) is true but (2) is false point?
(D) Statement (1) is false but (2) is true
(A) 3.50 (B) 7.00
(C) 8.40 (D) 5
36. Statement-1 : In general phenolphthalein is used
as an indictor for the titratin of weak acid (HA) against
strong base (NaOH). 40. What is the molar mass of HA?
Statement-2 : At equivalent point solution is basic. (A) 180 (B) 222
(A) Statement (1) is correct and statement (2) is (C) 282 (D) None of these
correct and statement (2) is correct explanation for
(1) Paragraph for Quesiton 41 to 44
(B) Statement (1) is correct and statement (2) is Potash alum is K Al(SO4)2. 12H2O. As a strong eletrolyte,
correct and statement (2) is NOT correct explanation it is considered to be 100% dissociated into K+, Al3+
for (1) and SO 42–. The solution is acidic because of the
(C) Statement (1) is true but (2) is false hydrolysis of Al3+, but not so acidic as might be
(D) Statement (1) is false but (2) is true expected, because the SO42– can sponge up some the
2+ H3O+ by forming HSO4–. Given a solution made by
37. Statement-1 : Moles of Sr of furnished by dissolving 11.85 gm of KAl(SO4)2.12H2O in enough water
sparingly soluble substance Sr(OH) decreases due to to make 100 cm3 of solution. What is [H3O+] of the
2
dilution in its saturated solution. solution if

394 - Rajeev Gandhi Nagar Kota, Ph. No. 0744-2209671, 93141-87482, 93527-21564
IVRS No. 0744-2439051, 0744-2439052, 0744-2439053 www.motioniitjee.com, email-hr.motioniitjee@gmail.com
IONIC EQUILIBRIUM Page # 23

41. None of the ion is hydrolysing Column-II (Exist b/w colour transition range of
(A) 10–7M (B) less than 10–7 M an indicator)
(C) More than 10–7M (D) 0.0 (P) Phenol Red (6.8 to 8.4)
(Q) Propyl red(4.6 to 6.4)
42. Only Al3+ is hydrolysing and its first hydrolsis (R) Phenolphthalein (8.3 to 10.1)
constant is 1.4 × 10–5 M (S) Malachite green (11.4 to 13)
(A) 1.87×10–3M (B) 6.24 ×10–4M
(C) 0.09 M (D) None of these

43. Only SO42– is hydrolysing and acid dissociation


constant of HSO4– in water is 1.25×10–2.
(A) 1.26 ×10–3M (B) 6.32 × 10–7 M
–8
(C) 1.58 × 10 M (D) None of these

44. Both Al3+ and SO42– are hydrolysing.


(A) 2.93 × 10–4 M (B) 0.0114 M
–6
(C) 5.43 × 10 M (D) None of these

Match The column


45. Column-I
 10 litre of 0.03N X(OH)2  strong diacidic base  
 
  
 5litre of 0.08 MHNO 
 3 
(A)   
 
 485litre of 0.01MNaNO3 
 

 10 ml of 0.5 MRNH3Cl Kh  109  


 
  
(B)  
 40 ml of 0.125 MKOH 
 
 100 ml of 0.8 MHCO3 
 
  
 100 ml of 0.4 M CO2  
(C)  3

 for H CO , use K  4  107 & K  4  1011 
 
 2 3 a 1 a  2

(D)Saturated aqueous solution of Co(OH)3(Ksp=2.7×10–43)


Column-II
(P) pH 3.7 (Q) pH 11
(R) pH 7 (S) pH 10

46. Column-I (pH of the resultant solution)


(A) 200 ml of H2SO4 solution (specific gravity 1.225
containing 25% H2SO4 by weight) + 800 ml of 0.525M
strong triacidic base X(OH)3
(B) 50 ml of 0.1 M HCO3– + 50 ml of 0.8 M CO32–
7 11
(H2CO3: K a  4  10 ,K a  2  10 )
1 2

(C) 50 ml of 0.2 M HA(aq)(Ka = 10–5) + 50 ml of 0.1 M


HCl(aq) + 100 ml of 0.13 M NaOH(aq)

394 - Rajeev Gandhi Nagar Kota, Ph. No. 0744-2209671, 93141-87482, 93527-21564
IVRS No. 0744-2439051, 0744-2439052, 0744-2439053 www.motioniitjee.com, email-hr.motioniitjee@gmail.com
Page # 24 IONIC EQUILIBRIUM

Exercise - II (One or more than one option correct)


1. Which of the following statement(s) is/are correct?
(A) the pH of 1.0 × 10–8 M solution of HCl is 8
(B) the conjugate base of H2PO4– is HPO42–
(C) autoprotolysis constant of water increases with
temperature
(D) When a soltion of a weak monoprotic acid is titrated
again a strong base, at half -neutralization point pH =
(1/2) pKa.

2. A 2.5 gm impure sample containing weak monoacidic


base (Mol. wt. = 45) is dissolved in 100 ml water and
th
1
titrated with 0.5 M HCl when   of the base was
5
neutralised the pH was found to be 9 and at equivalent
point pH of solution is 4.5. Given : All data at 25º C &
log 2 = 0.3.
Select correct statement(s).
(A) Kb of base is less than 10–6
(B) Concentration of salt (C) at equivalent point is 0.25 M
(C) Volume of HCl is used at equavalent point is 100 ml
(D) Weight percentage of base in given sample is 80%

3. Select incorrect statement(s).


(A) Phenolphthalein is suitable indicator for the
titraction of HCl(aq) with NH4OH (aq).
(B) An acid-base indicator in a buffer solution of pH =
pKln + 1 is ionized to the extent of 90%
(C) In the titration of a monoacidic weak base with a
strong acid, the pH at equivalent point is always
1
calculated by pH = [pKw – pKb – logC]
2
(D) When Na3PO4(aq) is titrated with HCl (aq.), the
pH of solution at second equivalent point is calculated
1
by [pKa + pKa ]
2 1 2

4. Which of the following is true for alkaline aqueous


solution?
pk w pk w
(A) pH > (B) pH > pOH (C) pOH < (D) pH < pOH
2 2

5. A buffer solution can be prepared from a mixture of


(A) sodium acetate and acetic acid in water
(B) sodium acetate and hydrochloric acid in water
(C) ammonia and ammonium chloride in water
(D) ammonia and sodium hydroxide in water.

394 - Rajeev Gandhi Nagar Kota, Ph. No. 0744-2209671, 93141-87482, 93527-21564
IVRS No. 0744-2439051, 0744-2439052, 0744-2439053 www.motioniitjee.com, email-hr.motioniitjee@gmail.com
IONIC EQUILIBRIUM Page # 25

Exercise - III Subjective Level-I


9. The solution of weak monoprotic acid which is 0.01
1. Calculate change in concentration of H+ ion in one M has pH = 3. Calculate Ka of weak acid.
litre of water, when temperature changes from 298 K
to 310 K. Given Kw (298) = 10–14 Kw (310) = 2.56 × 10–14
10. Boric acid is a weak monobasic acid. It ionizes in
water as
2. Calculate the number of H+ present in one ml of
solution whose pH is 13. B(OH)3 + H2O B(OH) 4– + H+ : Ka = 5.9 × 10–10
Calculate pH of 0.3 M boric acid.
3. pH of dilute solution of HCl is 6.95. Calculate molarity
of HCl solution 11. Calculate [H+] and [CHCl2COO–] in a solution that
is 0.01 M in HCl and 0.01 M in CHCl2COOH.
4. Calculate pH of following solutions : Take (Ka = 2.55 × 10–2).
(a) 0.1 M HCl (b) 0.1 M CH3COOH (Ka = 1.8 ×10–5)
(c) 0.1 M NH4OH (Ka = 1.8 ×10–5) 12. Calculate [H + ], [CH 3COO –] and [C 7H 5O 2–] in
(d) 0.1 M H2SO4 (50 ml) + 0.4 M HCl (50 ml) solution that is 0.02 M in acetic acid and 0.01 M in
benzoic acid. Ka (acetic) = 1.8 × 10–5, Ka (benzoic) =
(e) 10–8 M HCl (f) 10–10 M NaOH
6.4 × 10–5.
(g) Decimolar solution of Baryta (Ba(OH)2), diluted 100
times
13. Calculate pH of resultant solution of 0.1 M HA +
(h) 10–3 mole of KOH dissolved in 100 L of water.
0.1 M HB
(i) 0.5 M HCl (25 ml) + 0.5 M NaOH (10 ml) + 40 ml H2O
[Ka (HA) = 2 × 10–5 ; Ka(HB) = 4 × 10–5]
(j) Equal volume of HCl solution (PH = 4) + 0.0019 N
HCl solution
POLYPROTIC ACIDS & BASES
(k) 10-6 M CH3COOH (Ka = 1.8 × 10–5)
14. Calculate [H+], [H2PO4–], [HPO42–] and [PO43–] in
(l) 10-8 M CH3COOH (Ka = 1.8 × 10–5)
a 0.01 M solution of H3PO4.
Take K1 = 10–3, K2 = 10–8. K3 = 10–13.
5. Calculate :
(a) Ka for monobasic acid and whose 0.10 M solution
15. Calculate the pH of 0.1 M solution of H2NCH2 CH2
has pH of 4.50
NH2 ; ethylenediamine (en). Determine the en H22+.
(b) Kb for a monoacidic base whose 0.10 M solution
has a pH of 10.50. concentration in the solution. K b1 and K b2 values of
ethylenediamine are 8.5 × 10 –5 and 7.1 × 10 –8
respectively.
 2 
6. Calculate the ratio of degree of dissociation   
 1
16. What are the cocentrations of H+, HSO4–, SO42–
when 1 M acetic acid solution is diluted to 1/100 times.
and H2 SO4 in a 0.20 M solution of sulphuric acid ?
[Given Ka = 10–5M ]
Given : H2SO4  H+ + HSO4– ; strong

7. Calculate the ratio of degree of dissociation of HSO4– H+ + SO42– ; K2 = 10–2 M


acetic acid and hydrocyanic acid (HCN) in 1 M their
respective solution of acids.
17. What are the concentration of H + , H 2C 2O 4,
[Given K a( CH3 COOH)  18
.  10 –5
; K a(HCN)  6.2  10 –10
] HC2O4– and C2O42– in a 0.1 M solution of oxalic acid ?
[K1 = 10–2 M and K2 = 10–5 M]

8. The pH of aqueous solution of ammonia is 11.5.


Find molarity of solution. Kb (NH4OH) = 1.8 × 10–5.

394 - Rajeev Gandhi Nagar Kota, Ph. No. 0744-2209671, 93141-87482, 93527-21564
IVRS No. 0744-2439051, 0744-2439052, 0744-2439053 www.motioniitjee.com, email-hr.motioniitjee@gmail.com
Page # 26 IONIC EQUILIBRIUM

18. Nicotine, C10H14N2, has two basic nitrogen atoms 26. Calculate the pH of a solution made by mixing
and both can react with water to give a basic solution 50.0 ml of 0.2 M NH 4Cl & 75.0 ml of 0.1 M NaOH
[K b(NH 3) = 1.8 × 10 –5]
Nic(aq) + H2O(l) NicH+ (aq) + OH– (aq)

NicH+(aq) + H2O(l) NicH22+ (aq) + OH– (aq) INDICATORS


K b1 is 7 × 10 –7 and K b2 is 10 –10 . Calculate the 27. For the acid indicator thymol blue, pH is 2.0 when
approximate pH of 0.020 M solution. half the indicator is in unionised form. Find the % of
Sol. indicator in unionised form in the solution with [H+] =
19. H 3A is a weak triprotic acid (K a = 10 –5, K a = 4×10–3 M.
1 2
10 –9, K a = 10 –13
3
Calculate the value of pX of 0.1 M H3A(aq) solution 28. At what pH does an indicator change colour if
 A3   the indicator is a weak acid with Kind = 4 × 10–4 . For
  which one(s) of the following neutralizations would
where pX = – logX & X = .
HA2   the indicator be useful ? Explain.
 
(a) NaOH + CH3COOH (b) HCl + NH3 (c) HCl + NaOH
BUFFER SOLUTION
20. Determine [OH–] of 0.050 M solution of ammonia 29. What indicator should be used for the titration
to which sufficient NH4Cl has been added to make of 0.10 M KH2BO3 with 0.10 M HCl ?
.  10 –5 ]
the total [NH4+] equal to 0.100. [K b(NH3 )  18 Ka (H3BO3) = 7.2 × 10–10

30. Bromophenol blue is an acid indicator with a Ka


21. Calculate the pH of solution prepared by mixing value of 6 × 10–5. What % of this indicator is in its
50.0 mL of 0.200 M HC2H3O2 and 50.0 mL of 0.100 M basic form at a pH of 5 ?
NaOH. [Ka(CH3COOH) = 1.8 × 10–5]

31. An acid base indicator has a Ka of 3 × 10–5 . The


22. A buffer of pH 9.26 is made by dissolving x moles acid form of the indicator is red & the basic form is
of ammonium sulphate and 0.1 mole of ammonia into blue. By how much must the pH change in order to
100 mL solution. If pKb of ammonia is 4.74, calculate change the indicator form 75% red to 75% blue ?
value of x.

HYDROLYSIS
23. 50 mL of 0.1 M NaOH is added to 75 mL of 0.1 M 32. What is the OH– concentration of a 0.08 M
NH4Cl to make a basic buffer. If pKa of NH4+ is 9.26, solution of CH3COONa. [Ka(CH3COOH) = 1.8 × 10–5]
calculate pH.

33. Calculate the pH of a 2.0 M solution of NH4Cl.


24. (a) Determine the pH of a 0.2 M solution of [Kb(NH3) = 1.8 × 10–5]
pyridine C5H5N. Kb = 1.5 × 10–9
(b) Predict the effect of addition of pyridinium ion
34. 0.25 M solution of pyridinium chloride C5H6N+ Cl–
C5H5NH+ on the position of the equilibrium. Will the pH
was found to have a pH of 2.699. What is Kb for
be raised or lowered ?
pyridine, C5H5N ?
(c) Calculate the pH of 1.0 L of 0.10 M pyridine
solution to which 0.3 mol of pyridinium chloride
C5H5NH+Cl, has been added, assuming no change in 35. Calculate the extent of hydrolysis & the pH of
volume. 0.02 M CH3COONH4.
[Kb (NH3) = 1.8 × 10–5, Ka (CH3COOH) = 1.8 × 10–5]
25. Calculate the pH of a solution which results from
the mixing of 50.0 ml of 0.3 M HCl with 50.0 ml of 0.4 36. Calculate the percent hydrolysis in a 0.06 M
M NH3. [Kb(NH3) = 1.8 ×10–5] solution of KCN. [Ka(HCN) = 6 ×10–10]

394 - Rajeev Gandhi Nagar Kota, Ph. No. 0744-2209671, 93141-87482, 93527-21564
IVRS No. 0744-2439051, 0744-2439052, 0744-2439053 www.motioniitjee.com, email-hr.motioniitjee@gmail.com
IONIC EQUILIBRIUM Page # 27

has been added and 6.402 after 20.0 mL of NaOH has


37. Calculate the extent of hydrolysis of 0.005 M been added. What is the ionization constant of HX ?
K2CrO4. [K2 = 3.1 × 10–7 for H2CrO4]
(It is essentially strong for first ionization) 48. The equivalent point in a titration of 40.0 mL of
a solution of weak monoprotic acid occurs when 35.0
mL of a 0.10 M NaOH solution has been added. The pH
38. Calculate the percent hydrolysis in a 0.0100 M
of the solution is 5.75 after the addition of 20.0 mL of
solution of KCN. (Ka= 6.2 × 10–10)
NaOH solution. What is the dissociation constant of
the acid ?
39. A 0.010 M solution of PuO2(NO3)2 was found to
have a pH of 4.0. What is the hydrolysis constant, Kh,
49. A weak base (50.0 mL) was titrated with 0.1 M
for PuO22+, and what is Kb for PuO2 OH+ ?
HCl. The pH of the solution after the addition of 10.0
mL and 25.0 mL were found to be 9.84 and 9.24,
40. Calculate the pH of 1.0 × 10 –3 M sodium respectively. Calculate Kb of the base and pH at the
phenolate, NaOC6H5. for HOC6H5 is 1.05 × 10–10 equivalence point.

41. What is the pH of 0.1 M NaHCO 3 ? K 1 = 4.5 × 50. A weak acid (50.0 mL) was titrated with 0.1 M
10 –7, K 2 = 4.5 × 10 –11 for carbonic acids. NaOH. The pH values when 10.0 mL and 25.0 mL of
base have been added are found to be 4.16 and 4.76,
respectively. Calculate Ka of the acid and pH the
42. Calculate pH of 0.05 M potassium hydrogen
equivalence point.
phthalate, KHC8H4O4.

H2C8 H4O4 + H2O H3O+ + HC8H4O4– pK1 = 2.94


51. CH3COOH (50 ml, 0.1 M) is titrated against 0.1
HC8H4O4– + H2O H3O+ + C8H4O42– pK2 = 5.44 M NaOH solution. Calculate the pH at the addition of 0
ml, 10 ml, 20 ml, 25 ml, 40 ml, 50 ml of NaOH. Ka of
CH3COOH is 2 × 10–5.
43. The acid ionization (hydrolysis) constant of Zn2+
is 1.0 × 10–9
(a) Calculate the pH of a 0.001 M solution of ZnCl2 SOLUBILITY & SOLUBILITY PRODUCTS
(b) What is the basic dissociation constant of Zn(OH)+ ? 52. The values of Ksp for the slightly soluble salts MX
and QX2 are each equal to 4.0 × 10–18. Which salt is
more soluble ? Explain your answer fully.
ACID BASE REACTIONS & TITRATIONS
44. Calculate OH– concentration at the equivalent
53. The solubility of PbSO4 in water is 0.038 g/L.
point when a solution of 0.1 M acetic acid is titrated
Calculate the solubility product constant of PbSO4 in
with a solution of 0.1 M NaOH. Ka for the acid = 1.9 ×
water.
10–5.

45. Calculate the hydronium ion concentration and 54. How many mol Cul (Ksp = 5 × 10–12) will dissolve
pH at the equivalence point in the reaction of 22.0 mL in 1.0 L of 0.10 M Nal solution ?
of 0.10 M acetic acid, CH3COOH, with 22.0 mL of 0.10
M NaOH.(Ka = 1.8 × 10–5) 55. A solution of saturated CaF2 is found to contain
4.1 × 10–4 M fluoride ion. Calculate the Ksp of CaF2.
46. Calculate the hydronium ion concentration and Neglect hydrolysis.
the pH at the equivalence point in a titration of 50.0
mL of 0.40 M NH3 with 0.40 M HCl.(Kb = 1.8 × 10–5) 56. The solubility of ML2 (formula weight, 60 g/mol)
in water is 2.4 × 10–5 g/100 mL solution. Calculate
47. In the titration of a solution of a weak acid HX solubility product constant for ML2.
with NaOH, the pH is 5.8 after 10.0 mL of NaOH solution

394 - Rajeev Gandhi Nagar Kota, Ph. No. 0744-2209671, 93141-87482, 93527-21564
IVRS No. 0744-2439051, 0744-2439052, 0744-2439053 www.motioniitjee.com, email-hr.motioniitjee@gmail.com
Page # 28 IONIC EQUILIBRIUM

57. What is the solubility (in mol/L) of Fe(OH)3 in a 68. A recent investigation of the complexation of
solution of pH = 8.0 ? [Ksp for Fe(OH)3 = 1.0 × 10–36] SCN– with Fe3+ represented by constant K1, K2 and K3
as 130, 16, and 1.0 respectively. What is the overall
formation constant of Fe(SCN)3 from its component
58. Calculate the solubility of A2X3 in pure water,
ions, and what is the dissociation constant of Fe(SCN)3
assuming that neither kind of ion reacts with water. For
into its simplest ions on the basis of these data ?
A2X3, [Ksp = 1.1 × 10–23]

69. How much AgBr could dissolve in 1.0 L of 0.40 M


59. Determine the solubility of AgCl in 0.1 M BaCl2.
[Ksp for AgCl = 1 × 10–10] NH3 ? Assume that Ag(NH3)2+ is the only complex
formed.
[Kf(Ag(NH3)2+) = 1 × 108 ; Ksp (AgBr) = 5 × 10–13]
60. What mass of Pb2+ ion is left in solution when
50.0 mL of 0.20 M Pb(NO3)2 is added to 50.0 ml 1.5 M
NaCl
PROFICIENCY TEST
[Given Ksp for PbCl2 = 1.7 × 10–4]
1. True/False. When a solution of a weak monoprotic
61. A solution has a Mg2+ concentration of 0.0010 acid is titrated against a strong base, half-
mol/L. Will Mg(OH)2 precipitate if the OH– concentration 1
neutralization point, pH = pK a
of the solution is [Ksp = 1.2 × 10–11] 2
(a) 10–5 mol/L (b) 10–3 mol/L ? 2. True/False. A solution of sodium acetate and
ammonium acetate can act as a buffer.
62. Calculate solubility of PbI2 (Ksp = 1.4 × 10–8) in
water at 25°, which is 90% dissociated 3. True/False. If the solubility of the salt Li3Na3(AlF6)2
is x, then its solubility product would be 2916 x8.

63. 500 mL of 0.01 AgNO3 is mixed with 250 ml each 4. True/False. A buffer has maximum buffer capacity
of NaBr and NaCl, each having molarity 0.02 M. Find when the ratio of salt to acid is 10.
equilibrium concentrtion of Br– (moles/L).
Given : Ksp(AgBr) = 5 × 10–13, Ksp(AgCl) = 10–10. 5. True/False. In the presence of a common ion
(incapable of froming complex ion), the solubility of
salt decrease.
64. Calculate solubility of AgCN (Ksp = 4 × 10–16) in a
buffer solution of PH = 3. 6. In a mixture of waek acid and its salt, the ratio of
concentration of salt to acid is increased ten fold.
SIMULTANEOUS SOLUBILITY The pH of the solution would _______________ by
65. Calculate the Simulataneous solubility of AgSCN ________________ unit.
and AgBr. Ksp (AgSCN) = 1.1 × 10–12, Ksp(AgBr) = 5 ×
7. The solubility of CH3COOAg in water considering
10–13.
hydrolysis of CH3COO– ions would be _________ than
that ignoring the hydrolysis.
66. Calculate F– in a solution saturated with respect
of both MgF2 and SrF2. Ksp (MgF2) =9.5 × 10–9, 8. From an equimolar solution of Cl– and Br– ions, the
Ksp(SrF2) = 4 × 10–9. addition of Ag + will selectively precipitates
COMPLEXATION EQUILIBRIA ______________
67. Assuming no change in volume, calculate the (Ksp of AgCl & AgBr are 1 × 10 –10 & 1 × 10 –13
minimum mass of NaCl necessary to dissolve 0.010
respectively).
mol AgCl in 100 L solution :
[K1(AgCl2–) = 3 × 105, Ksp = (AgCl) = 1 × 10–10] 9. The solubility of AgCl in NH3 is __________ than

394 - Rajeev Gandhi Nagar Kota, Ph. No. 0744-2209671, 93141-87482, 93527-21564
IVRS No. 0744-2439051, 0744-2439052, 0744-2439053 www.motioniitjee.com, email-hr.motioniitjee@gmail.com
IONIC EQUILIBRIUM Page # 29

the solubility in pure water because of complex ion, ___________ than 7.


+
[Ag(NH3)2] formation.
21. In the reaction I 2 + I –  I 3– , I 2 acts as
10. The hydrolytic constant Kh for the hydrolytic __________.
equilibrium
22. An equimolar solution of NaNO2 and HNO2 can
– – –12
H2PO + H2O  H3PO4 + OH is 1.4 × 10
4
act as a _________ solution.

What is the value of ionization constant for the H3PO4 23. Larger the value of pKa, ____________ is the
– +
+ H2O  H2PO + H3O ?
4
acid.

11. Given the equilibrium constants 24. Between Na+ & Ag+ ion, ______________ is a
stronger Lewis acid.
HgCl+ + Cl– HgCl2 ; K1 = 3 × 106

HgCl2 + Cl– HgCl3– ; K2 = 8.9 25. Salts of strong acid and weak bases undergo
___________ hydrolysis.
The equilibrium constant for the dispropotionation
equilibrium. 26. For salt of weak acid with weak bass, degree of
hydrolysis is ____________ of concentration of the
2HgCl2 HgCl+ + HgCl3– is salt in solution.

12. Under which set of conditions is the ionic product BEWARE OF SNAKES
of water, Kw, constant at a given temperature aqueous
1. General Mistake : pH of a neutral water solution is
system ?
always equal to 7.
13. If the salts M 2X, QY 2 and PZ 3 have same
Explanation : pH of neutral water depend on
solubilities (<<< 1), thier Ksp values are related as
____________ pK w
temperature. Since pH (neutral point) = ; pK w
2
14. Ka for an acid HA is 1 × 10–6. Kb for A– would be
____________. decrease with temperature hence pH of neutral

15. An equeous solutino of K2SO4 has pH nearly equal solution.

to __________. 2. General Mistake : If a solutiion is dilute half time

16. The pH of solution which is 0.1 M in sodium pH of solution becomes double.

acetate and 0.01 M in acetic acid (pKa = 4.74) would Explanation : Infact pH increases by 0.3010 unit. If
be ______ it is diluted x times pH increases by log x.

17. The conjugate acid of sulphate (SO 42– ) is e.g. If solution is diluted 10 times pH increases by
__________. log1010 = 1 unit.

18. The value of Kw __________ with increase in 3. General Mistake : For calculation of pH of 10–6 M
temperature.
CH3COOH the formual (H+) = K a c will give
19. AgCl is _________ soluble in aqueous sodium
chloride solution than in pure water.
 .  10 –5  10 –6 
pH = – log  18  = 5.37
20. The buffer HCOOH/HCOONa will have pH

394 - Rajeev Gandhi Nagar Kota, Ph. No. 0744-2209671, 93141-87482, 93527-21564
IVRS No. 0744-2439051, 0744-2439052, 0744-2439053 www.motioniitjee.com, email-hr.motioniitjee@gmail.com
Page # 30 IONIC EQUILIBRIUM

Explanation : 5.37 is incorrect answer. pH shold be solution to which enough HCl has been added to
produce a [H+] of 2 × 10–4.
–K a  K 2a  4K a c
calculated by taking  
2c
[H ]2 [S2– ] (2  10 –4 ) 2 [S2 – ]
Sol. K1K2 = = = 1.0 × 10–21
[H2 S] 0.10
4. Genral Mistake : If 103 mole CH3COONa and 1 mole
CH3COOH is added in 104 litres water the pH of resulting or

10 3
solution is equal to pH = pKa + log = 7.74 .  10 –22
10
1 [S2–] = = 2.5 × 10–15
4  10 –8

Explanation : 7.74 is incorrect answer. The CH3COOH


concentration is too low to be taken as constituent
of buffer solutio. Use salt hydrolysis formula instead
to calculated the pH.

5. General Mistake : The equilibrium concentration


of anion and cation of a sparingly soluble salt (A2C3)
and a and c mole lit–1 respectively. The solubility
product is (2a2) (3c)3 = Ksp

Explanation : Ksp = a2c3.

6. General Mistake : pH of 10–8 M HCl is equal to 8.

Explaination : pH = 8 means basic solution.


Contribution of water can not be neglected in this
case.

7. General Mistake : If NaOH is added to NH4Cl so


that NaOH is limiting, the resulting solutio is containing
some remaining conc. of NH4Cl. Now use salt hydrolysis
condition to calculate pH of solution.

Explanation : The addition of NaOH in NH4Cl results


in basic buffer solution.

8. General Mistake : Do not use the K1 K2 from of


equation unless you have an independent method of
calculating [H+] or [S2–]

Explanation : Determine the [S2–] in a saturated H2S

394 - Rajeev Gandhi Nagar Kota, Ph. No. 0744-2209671, 93141-87482, 93527-21564
IVRS No. 0744-2439051, 0744-2439052, 0744-2439053 www.motioniitjee.com, email-hr.motioniitjee@gmail.com
IONIC EQUILIBRIUM Page # 31

Exercise - IV Subjective Level-II

1. Calculate the percent error in the [H 3 O + ] 9. Mixtures of solution. Calculate the pH of the
concentration made by neglecting the ionization of following solution. (Use data of Q.12)
water in 10–6 M NaOH solution. (A) 40 ml of 0.050 M Na2CO3 + 50 ml of 0.040 M HCl ;
(B) 40 ml of 0.020 M Na3PO4 + 40 ml of 0.040 M HCl ;
2. A solution contains HCl, Cl2HC COOH & CH3COOH at (C) 50 ml of 0.10 M Na3 PO4 + 50 ml of 0.10 M NaH2PO4
concentation 0.09 M in HCl, 0.09 M in Cl2HC COOH &
(D) 40 ml of 0.10 M H3PO4 + 40 ml of 0.10 M Na3 PO4.
0.1 M in CH3COOH. pH for the solution is 1. Ionization
constant of CH3COOH = 105. What is the magnitude
of K for dichloroacetic acid ? 10. The electrolytic reduction of an organic nitro
compound was carried out in a solution buffered by
acetic acid and sodium acetate. The reaction was
3. A solution of chloroacetic acid, ClCH 2 COOH
containing 9.45 grams in 500 ml of the solution has a RHO2 + 4H3O+ + 4e  RNHOH + 5H2O
pH of 2.0. What is the degree of ionization the acid. 300 ml of 0.0100 M solution of RNO2 buffered initially
at pH 5.00 was reduced, with the reaction above going
to completion. The total acetate concentration, [HOAc]
4. The Kw of water at two different temperature is :
+ [OAc–], was 0.50 M. Calculate the pH of the solution
T 25°C 50°C after the reduction is complete.
Kw 1.08 × 10–14 5.474 × 10–14
Assuming that H of any reaction is independent of 11. It is desired to prepare 100 ml of a buffer of pH
temperature, calculate the enthalpy of neutralization 5.00. Acetic, benzoic and formic acids and their salt
of strong acid and strong base. are availble for use. Which acid should be used for
maximum effectiveness against increase in pH ? What
acid-salt ratio should be used ? pKa values of these
5. What is the pH of 1.0 M solution of acetic acid? To acids are : acetic 4.74; benzoic 4.18 and formic 3.68.
what volume must 1 litre of the solution be dilute so
that the pH of the resulting solution will be twice the
original value. Given Ka = 1.8 × 10–5. 12. Calculate the pH of 0.1 M solution of (i) NaHCO3,
(ii) Na2HPO4 and (iii) NaH2PO4. Given that ;

6. A handbook states that the solubility of methylamine CO2 + H2O H+ + HCO3– ; K1 = 4.2 × 10–7 M
CH3NH2(g) in water at 1 atm pressure at 25° Cis 959
volumes of CH3NH2 (g) per volume of water (pkb = HCO3– H+ + CO32– ; K2 = 4.8 × 10–11 M
3.39)
H3PO4 H+ + H2PO4– ; K1 = 7.5 × 10–3 M
(a) Estimate the max. pH that can be attained by
dissolving methylamine in water. H2PO4– H+ + HPO42– ; K2 = 6.2 × 10–8 M
(b) What molarity NaOH (aq.) would be required to
yield the same pH ? HPO42– H+ + PO43– ; K3 = 1.0 × 10–12 M
7. The equilibrium constant of the reaction. 13. When a 40 mL of a 0.1 M weak base in titrated
with 0.16 M HCl, the pH of the solution at the end
2Ag(s) + 2I– + 2H2O 2Agl(s) + H2(g) + 2OH– point is 5.23. What will be the pH if 15 mL of 0.12 M
is 1.2 × 10–23 at 25°C. Calculate the pH of a solution NaOH is added to the resulting solution.
at equilibrium with the iodine ion concentation = 0.10 14. A bufer solution was prepared by dissolving 0.05
and the pressure of H2 gas = 0.60 atm. mol formic acid & 0.06 mol sodium formate in enough
8. Mixture of solutions. Calculate the pH of the water to make 1.0 L of solution. Ka for formic acid is
following solutions. (Use data of Q.12) 1.80 × 10–4

(A) 50 ml of 0.12 M H3PO4 + 20 ml of 0.15 M NaOH ; (a) Calculate the pH of the solution.

(B) 50 ml of 0.12 M H3PO4 + 40 ml of 0.15 M NaOH ; (b) If this solution were diluted to 10 times its volume,
what would be the pH ?
(C) 40 ml of 0.12 M H3PO4 + 40 ml of 0.18 M NaOH ;
(c) If the solution in (b) were diluted to 10 times its
(D) 40 ml of 0.10 M H3PO4 + 40 ml of 0.25 M NaOH. volume, what would be the pH ?

394 - Rajeev Gandhi Nagar Kota, Ph. No. 0744-2209671, 93141-87482, 93527-21564
IVRS No. 0744-2439051, 0744-2439052, 0744-2439053 www.motioniitjee.com, email-hr.motioniitjee@gmail.com
Page # 32 IONIC EQUILIBRIUM

15. How many moles of sodium hydroxide can be Enough strong base was now added (6m eq.) to
added to 1.00 L of a solution 0.1 M in NH3 & 0.1 M in completely convert the salt. The total volume was 50
NH4Cl without changing the pOH by mor than 1.00 ml. Find the pH at this point.
unit ? Assume no change in volume. Kb(NH3) = 1.8 ×
10–5.
24. An organic monoprotic acid [0.1M] is titrated
against 0.1 M NaOH. By how much does the pH change
16. 20 ml of a solution of 0.1 M CH3COOH solution is between one fourth and three fourth stages of
being titrated against 0.1 M NaOH solution. The pH neutralization ? If at one third stage of neutralization,
vaues after the addition of 1 ml & 19 ml of NaOH are the pH is 4.45 what is the dissociation constant of
(pH)1 & (pH)2, what is pH ? the acid? Between what stages of neutralisation may
the pH change by 2 units ?

17. Calculate the OH– concentration and the H3PO4


concentration of a solution prepared by dissolving 0.1 25. At 25°C, will a precipitate of Mg(OH)2 form in a
mol of Na3PO4 in sufficient water to make 1 l of solution. 10–4 M solution of Mg(NO3)2 if pH of the solution is
K1 = 7.1 × 10–3, K2 = 6.3 × 10–8, K3 = 4.5 × 10–13. adjusted to 9.0. Ksp [Mg(OH)2] = 10–11 M3. At what min
value of pH will precipitation start.

18. Calculate the value sof the equilibrium constants


for the reactions with water of H2PO4–, HPO42–, and 26. The salt Zn(OH)2 is involved in the following two
PO43– as bases, Comparing the relative values of the equilibria,
two equilibrium constants of H2PO4– with water, deduce
whether solutions of this ion in water are acidic or Zn(OH)2 (s) Zn2+ (aq) + 2OH– (aq) ; K sp =
basic. Deduce whether solutions of HPO42– are acidic 1.2 × 10–17
or basic.
Zn(OH)2 (s) + 2OH– (aq) [Zn(OH)4]2– (aq.) ;
Take K1 = 5 × 10–3, K2 = 5 × 10–8, K3 = 5 × 10–13, Kc = 0.13
Calculate the pH of solution at which solubility is
19. If 0.0050 mol NaHCO3 is added to 1 litre of a minimum.
buffered solution at pH 8.00, how much material will
exist in each of the three forms H2CO3, HCO3– and
CO32– ? For H2CO3, K1 = 5 × 10–7, K2 = 5 × 10–13. 27. What is the solubility of AgCl in 0.20 M NH3 ?
Given : Ksp(AgCl) = 1.7 × 10–10 M2, K1 = [Ag(NH3)+] /
[Ag+] [NH3] = 2.33 × 103 M–1 and
20. Equilibrium constant for the acid ionization of
Fe3+ to Fe(OH)+2 and H+ is 6.5 × 10–3. What is the K2 = [Ag(NH3)2+] / [Ag (NH3)+ [NH3] = 7.14 × 103 M–1
max. pH, which could be used so that at least 95% of
the total Fe3+ in a dilute solution. exists are Fe3+.
28. Predict whether or not AgCl will be precipitated
21. The indicator phenol red is half in the ioic form from a solution which is 0.02 M in NaCl and 0.05 M in
when pH is 7.2. If the ratio of the undissociated form KAg(CN)2. Given Kinst(Ag(CN)2–) = 4.0 × 10–19 M2 and
to the ionic form is 1 : 5, find the pH of the solution. Ksp(AgCl) = 2.8 × 10–10 M2.
With the same pH for solution, if indicator is altered
such that the ratio of undissociated form to dissociated
form becomes 1 : 4, find the pH when 50% of the new 29. Equal volumes of 0.02 M AgNO3 and 0.02 M HCN
indicator is in ionic form. were mixed. Calculate [Ag+] at equilibrium. Take
Ka(HCN) = 9 × 10–10, Ksp (AgCN) =4 × 10–6.

22. A solution of weak acid HA was titrated with


base NaOH. The equivalence point was reaced when 30. Show that solubility of a sparingly soluble salt
36.12 ml of 0.1 M NaOH has been added. Now 18.06 M2+ A2– in which A2– ions undergoes hydrolysis is given
ml of 0.1 M HCl were added to titration solution, the by
pH was found to be 4.92. What will be the pH of the
solution obtained by mixing 10 ml 0.2 M NaOH and 10
 [H ] [H ] 2 
S  K sp  1   
ml of 0.2 M HA.  K2 K 1K 2 

where K1 and K2 are the dissociation constant of acid


23. A weak base BOH was titrated against a strong H2A. Ksp is solubility product of MA.
acid. The pH at 1/4 the equivalence point was 9.24.

394 - Rajeev Gandhi Nagar Kota, Ph. No. 0744-2209671, 93141-87482, 93527-21564
IVRS No. 0744-2439051, 0744-2439052, 0744-2439053 www.motioniitjee.com, email-hr.motioniitjee@gmail.com
IONIC EQUILIBRIUM Page # 33

Exercise - V JEE-Problems
1. What will be the resultant pH when 200 ml aqueous 11. CH3NH2 (0.1 mole, Kb = 5 × 10–4) is added to
solution of HCl (pH = 2.0) is mixed with 300 ml of an 0.08 moles of HCl and the solution is diluted to one
aqueous solution of NaOH (pH=12.0)? [JEE 1998] litre, resulting hydrogen ion concentration is
(A) 1.6 × 10–11 (B) 8 × 10–11 [JEE 2005]
2. The solubility of Pb(OH)2 in water is 6.7 × 10–6 M. (C) 5 × 10 –5
(D) 2 × 10–2
Calculate the solubility of Pb(OH)2 in a buffer solution
of pH= 8. [JEE '1999]
12. Ag+ + NH3 [Ag(NH3)]+ ; k1 = 3.5 × 10–3
[JEE 2006]
3. The pH of 0.1 M solution of the following salts
increases in the order [JEE 1999] [Ag(NH3)]+ + NH3 [Ag(NH3)2]+ ;
(A) NaCl < NH4Cl < NaCN < HCl k2 = 1.7 × 10–3
(B) HCl < NH4Cl < NaCl < NaCN then the formation constant of [Ag(NH3)2]+ is
(C) NaCN < NH4Cl < NaCl < HCl (A) 6.08 × 10–6 (B) 6.08 × 106
(D) HCl < NaCl < NaCN < NH4Cl (C) 6.08 × 10–9 (D) None of these

4. An aqueous soltion of 6.3 g oxalic acid dihydrate is 13. The species present in solution when CO2 is
made up to 250 mL. The volume of 0.1 N NaoH required dissolved in water: [JEE 2006]
to completely neutralise 10 mL of this solution is : (A) CO2, H2CO3, HCO3–, CO32–
[JEE 2001] (B) H2CO3, CO32–
(A) 40 mL (B) 20 mL (C) 10 mL (D) 4 mL (C) CO32–, HCO3–
(D) CO2, H2CO3
5. For sparingly soluble salt ApBq, the relationship of
its solubility product (Ls) with its solubility (S) is 14. Solubility product constant (Ksp) of salts of types
[JEE 2001] MX, MX2 and M3X at temperature 'T' are 4.0 × 10–8,
(A) Ls = Sp + q, pp. qq (B) Ls = Sp + q, pp. qp 3.2 × 10–14 and 2.7 × 10–15, respectively. Solubilities
(C) Ls = Sp q, pp. qq (D) Ls = Sp q, (p.q)p + q (mol. dm–3) of the salts at temperature 'T' are in the
order [JEE 2008]
6. 500 ml of 0.2 M aqueous solution of acetic acid is (A) MX > MX2 > M3X (B) M3X > MX2 > MX
mixed with 500 mL of 0.2 M HCl at 25°C (C) MX2 > M3X > MX (D) MX > M3X > MX2
(a) Calculate the degree of dissociation of acetic acid
in the resulting solution and pH of the solution. 2
(b) If 6 g of NaOH is added to the above solution, 15. 2.5 mL of M weak monoacidic base
5
determine final pH. Assume there is no change in volume 2
on mixing. Ka of acetic acid is 1.75 × 10–5 M. (Kb = 1 × 10–12 at 25°C) is titrated with M HCl in
15
[JEE 2002] +
water at 25°C. The concentration of H at equivalence
point is (Kw = 1 × 10–4 at 25°C) [JEE 2008]
7. A solution which is 10–3 M each in Mn2+, Fe2+, Zn2+ (A) 3.7 × 10–13 M (B) 3.2 × 10– 7M
and Hg2+ is treated with 10–16 M sulphide ion. If Ksp, (C) 3.2 × 10–2 M (D) 2.7 × 10–2 M
MnS, FeS, ZnS and HgS are 10–15, 10–23, 10–20 and 10–
54
respectively, which one will precipitate first ? 16. The dissociation constant of a substituted
[JEE 2003] benzoic acid at 25ºC is 1.0 × 10–4. The pH of a 0.01 M
(A) FeS (B) MnS (C) HgS (D) ZnS solution of its sodium salt is : [JEE 2009]

8. Will the pH of water be same at 4°C and 25°C ? 17. Aqueous solutions of HNO3, KOH, CH3COOH, and
Explain. [JEE 2003] CH3COONa of identical cocentrations are provided. The
pair(s) of solutions which form a buffer upon mixing
9. HX is a weak acid (Ka = 10–5). It forms a salt NaX is(are). [JEE 2010]
(0.1 M) on reacting with caustic soda. The degree of (A) HNO3 and CH3COOH (B) KOH and CH3COONa
hydrolysis of NaX is [JEE 2004] (C) HNO3 and CH3COONa (D) CH3COOH and CH3COONa
(A) 0.01 % (B) 0.0001%
(C) 0.1% (D) 0.5% 18. In 1 L saturated solution AgCl [Ksp(AgCl) = 1.6
× 10–10], 0.1 mol of CuCl [Ksp(CuCl) = 1.0 × 10–6] is
10. 0.1 M of HA is titrated with 0.1 M NaOH, calculate added. The resultant concentration of Ag+ in the
the pH at end point. Given Ka(HA) = 5 × 10–6 and  << 1. solution is 1.6 × 10–x. The value of "x" is[JEE 2011]
[JEE 2003]

394 - Rajeev Gandhi Nagar Kota, Ph. No. 0744-2209671, 93141-87482, 93527-21564
IVRS No. 0744-2439051, 0744-2439052, 0744-2439053 www.motioniitjee.com, email-hr.motioniitjee@gmail.com
IONIC EQUILIBRIUM Page # 35

ANSWER-KEY
Exercise-I

1. A 2. C 3. C 4. B 5. B 6. C 7. B 8. D

9. B 10. A 11. D 12. D 13. B 14. B 15. D 16. D

17. C 18. D 19. C 20. C 21. B 22. A 23. D 24. D

25. D 26. A 27. C 28. B 29. A 30. A 31. C 32. D

33. B 34. A 35. A 36. B 37. D 38. C 39. C 40. C

41. A 42. A 43. C 44. A 45. A–P, B-Q, C–S, D–R 46. A–S, B–S, C-Q

Exercise-II

1. B,C 2. B,C 3. A,B,C 4. A,B,C 5. A,B,C

Exercise-III

1. 0.6 × 10–7 2. 6.022×107 3. 2.31 × 10–8

4. (a) 1 (b) 2.87 (c) 11.13 (d) 0.522 (e) 6.97 (f) 7 (g) 11.30 (h) 9 (i) 1 (j) 3 (k) 6.01 (l) 6.97 5. (a) Ka = 10–8 (b) Kb = 10–6

6. 10 7. 170.4 8. 0.556 9. 1.11 × 10–4 10. 4.87

11. [H+] = 1.612 × 10–2 M, [CHCl2COO–] = 6.126 × 10–3M 12. [H+] = 10–3 M, [CH3COO–] = 3.6 × 10–4 M, [C7H5O2–] = 6.4 × 10–4 M

13. 2.61 14. [H+] = [H2PO4–] = 2.7 × 10–3, [HPO42–] = 10–8, [PO43–] = 3.7 × 10–19 15. pH = 11.46, [enH22+] = 7.1 ×10–8

16. 0.2116 M, 0.1884 M, 0.0116 M, 0 17. 0.027M, 0.073 M, 0.027M, 10–5M 18. 10.07

19. 10 20. 9.0 × 10–6 21. 4.74 22. 0.05 mol 23. 9.56

24. (a) pH = 9.239 (b) Lowered (c) pH = 4.699 25. 8.7782 26. 9.7324

27. [HIn] = 28.57 % 28. (b) & (c) 29. Methyl red, one with pH = 5.22 as mid point of colour change

30. 85.71 % 31. pH = 0.954 32. [OH–] = 6.664 × 10–6 33. pH = 4.477

394 - Rajeev Gandhi Nagar Kota, Ph. No. 0744-2209671, 93141-87482, 93527-21564
IVRS No. 0744-2439051, 0744-2439052, 0744-2439053 www.motioniitjee.com, email-hr.motioniitjee@gmail.com
Solutions Slot – 2 (Chemistry) Page # 1

IONIC EQUILIBRIUM
EXERCISE – I SINGLE CORRECT (OBJECTIVE QUESTIONS)

1. NH2– + H+  NH3


[H ]2
Conjugate K1 = [H+] = K1[H3PO 4 ]
 H3PO4 
Acid
2. Amphiprotic : can accept and X.Y
K2 = =Y
Release H+ X
Only H2PO4– & HCO3– Y = [HPO42–] = K2

3. NaCl Solution : pH is the, pH


of water. 9. % Hydrolysis does not
depend on the conc. in case
As T , Kw , & [H+]  of “Weak acid + weak base : Salt”
pH at 25°C <7
10. Weak acid + Strong base : Salt
4. Volume of resulting solution = 100 ml
Kw/Ka
0.1  1 CN– + H2O HCN + OH–
[H+] = = 10–3
100 1
pH = 3. –x
x X X
80

Kw 10–14 X2
 1  1 = =
10   40 
Ka 1.3  10– 9 (1 / 80 – X)
 200

200 
5. [H+] =  2 X can be neglected
50 X = 3.1 × 10–4
[H+] = 10–2 3.1  10–4
pH = 2. % Hydrolysis = (1 ) × 100
80
6. HF H+ + F– = 2.48 %
pKw = pKa + pKb
[For conjugate Acid-Base] 11. Salt of weak acid & strong
 pKa = 14 – 10.87 = 3.17 Base
Ka = 6.76 × 10–4 CH3COONa CH3COO– + Na+
CH3COO– + H2O CH3COOH + OH–
+ –
7. HA H +A Basic Solution
1–x x x
x = 1% 12. Check Every option.
[H+] = 0.01
pH = 2 13. Salt formed : NH4Cl = 0.1 N
Solution will be slightly acidic due to
Hydrolysis
8. H3PO4 K1 H
+
H2PO4
X  Y Z xy
C–X 14. Ka = 1.8 × 10–5
Kw/Ka
H2PO4 K2 H 2
CH3COO– + H2O CH3COOH + OH–
+ HPO 4
X–Y X  Y Z Y Z
Given reaction is reverse of above
2– 3
HPO K3 H 
PO Ka
4 + 4
Y–Z X  Y Z Z Equilibrium const. = K
w
X >> Y >> Z
[H+] [H2PO4–] = X = 1.8 × 109

394 - Rajeev Gandhi Nagar Kota, Ph. No. 0744-2209671, 93141-87482, 93527-21564
IVRS No. 0744-2439051, 0744-2439052, 0744-2439053 www.motioniitjee.com, email-info@motioniitjee.com
Page # 2 Solutions Slot – 2 (Chemistry)

15. m. equivalent of KOH = 8 20. Salt of weak acid & weak base
m. equivalent of HCOOH = 16
Remaining m. eq. (HCOOH) = 8 K w .K a
[H+] =
Formed m. eq. (HCOOK) = 8 Kb
Acidic Buffer
pH = pKa = 4 – log2 pH = 12 [pKw + pKa – pKb]
= 3.7
pOH = 10.3 = 12 [14 + 4.8 – 4.78]
pH = 7.01
16. [H+]1 = 10–2 ; [H+]2 = 10–6

[H ]1 21. NaX + HCl NaCl + HX


 = 104 [HX] at equivalent point = 0.05
[H ]2
10–14
HX H+ + X– Ka =
17. H2A H+ + HA– Kb
0.05 – x x x Ka = 10–8
[H ][HA – ]
10–5 = x2
[H2 A]
10–8 = (Neglect x)
0.05 – x
HA– H+ + A2–
x = 5 × 10–5 pH = 4.65
[H ][A 2– ]
5 × 10–10 = Range of Indicator = 3 to 5
[A – ]
H2A 2H+ + A– Salt
22. pH = pKa + log
 2
[H ] [A ] 2– Acid
Koverall =
[H2 A] Salt
 5.5 = 5 + log
–10
= 5 × 10 × 10 –5 Acid
= 5 × 10–15
Salt
 = 3.16
Acid
18. RNH2 + H2O RNH+3 + OH–
Suppose x m. mole NaOH was added
0.01 – X X X+10–4
Acid + Base Salt
x(x  10–4 ) 10 x 10
= 2 × 10–6 (Neglect x) 10 - x 0 10 + x
0.01 – x
x2 + 10–4 x – 2 × 10–8 = 0 10  x
 = 3.16
x = 10–4 10  x
[OH–] = x + 10–4 x = 5.2 m.mole
= 2 × 10–4
5 .2
NaoH (mass) = × 40
19. pH = 8.7 pOH = 5.3 1000
Basic Buffer = 0.208 g
[Salt]
pOH = pKb + log
[Base] 23. A 2 X3 2A3+ + 3X2–
2y 3y
Salt Ksp = (2y)2 (3y)3
 =4 Ksp = 108y5
Base
If volume of salt = V ml
24. HgSO4 Hg2+ + SO42–
V  0 .2
 =4 S S
100  0.1
Ksp = S2
 V  200 ml 6.4 × 10–5 =S2
S = 8 × 10–3 mole/L
S = 8 × 10–3 × 103 mole/m3

394 - Rajeev Gandhi Nagar Kota, Ph. No. 0744-2209671, 93141-87482, 93527-21564
IVRS No. 0744-2439051, 0744-2439052, 0744-2439053 www.motioniitjee.com, email-info@motioniitjee.com
Solutions Slot – 2 (Chemistry) Page # 3

S = 8 mole/m3
[M2  ][H2S]
Qeq. =
25. Calculate the solubility ‘s’ for each option, [H ]2
Higher the value of ‘s’ Higher the solubility.
[0.01][0.1]
Qeq. = = 10–3
26. NaCl + HCl : Not the Buffer 1
and Solution is acidic due to HCl. Only PbS will ppt (Keq = 3 × 10–7)
pH < 7.
33. Precipitate started when
Salt Qsp > Ksp
27. pH = pKa + log Qsp = [Ba2+] [CO32–]
Acid
50% ionised [Salt] = [Acid] Qsp = [Ba2+] [10–4] > 5.1 × 10–9
pH = pKa = 4.5 [Ba2+] > 5.1 × 10–5
pOH = 9.5
34. MX4 M4+ + 4X–
28. For ppt Qsp > Ksp S 4S
CaF2  Ca2++2F– Ksp = [S][4S]4
Qsp = (Ca ) (F–)2
2+
Ksp = 256S5
(A) Qsp = 12.5 × 10–14 1/5
(B) Qsp = 12.5 × 10–10  K sp 
(C) Qsp = 12.5 × 10–13 S =  
 256 
(D) Qsp = 12.5 × 10–15
Only (B) option will get precipitate. 35.

36.
29. AgA Ag+ + A–
S S–X 37.
Kw
A– + H2O HA + OH– : Ka 38. From the Graph
S–X X X X = 10–5 19 ml
10–10
10–4 = S–X = 10–6 39. From the Graph
SX
pH 8.5
Ksp = S (S–X) = 11 × 10–6 × 10–6
= 1.1 × 10–11
1.2 0.222  19
40. =
30. Same as problem Number = 28 Mol. mass 1000
Mol. mass = 284.5
31. Qsp (AgBr) = [Ag+] [Br–]
10 5   2  10 7 
11.85
Qsp =  100   100  41. Mole of Potash Alum =
    474
–16
= 2 × 10 = 0.025
Qsp < Ksp Mole [Al3+] = 0.025
No precipitation Mole [K+] = 0.025
If none of the Ion Hydrolysed
10 5
[Ag+] = = 10–7 M [H+] = 10–7
100

32. For ppt Qsp > Ksp 42. Al3+ + H2O Kw


Al(OH)2+ + H+
MS + 2H+ M2+ + H2S Keq. Ka

+ 2– C-x x x
H2S 2H + S Ka1.Ka2
____________________________ 0.025
C= × 1000 = 0.25
MS M +S 2+ 2–
Ksp 100
Ksp = Keq. Ka1.Ka2
for ppt. Qeq. > Keq.

394 - Rajeev Gandhi Nagar Kota, Ph. No. 0744-2209671, 93141-87482, 93527-21564
IVRS No. 0744-2439051, 0744-2439052, 0744-2439053 www.motioniitjee.com, email-info@motioniitjee.com
Page # 4 Solutions Slot – 2 (Chemistry)

x2 Kw
 = 1.4 × 10–5 x2 = Kb = –5
0.25  x 0.1  x K x = 10
x2 = 1.4 × 0.25 × 10–5 x = 10–3 pOH = 3 ; pH = 11
x = 1.87 × 10–3
(C)Buffer : HCO3– / CO32–
43. SO42– + H2O Kw
HSO4– + OH– Salt
pH = pKa + log
Ka Acid
0.5-x x x 4
2 –14 = 11 – log 4 + log
x 10 8
= (Neglect x)
0.5  x 1.25  10–2 = 11 – log 4 – log 2
[OH–] = x = 6.32 × 10–7 = 11 – 3 log 2 =11 – 0.9
= 10.1
10–14
[H+] = = 1.58 × 10–8
6.25  10–7 (D) CO(OH)3 CO3+ + 3OH–
S 3S
27S4 = 27 × 10–44
44. Al3+ + SO42– + H2O Kw
Al(OH)2+ + HSO4– S = 10–11 &
K a.Kb [OH–] = [10–11 + x] : x from water
0.25-x 0.5-x x x [H+] = x & x (10–11 + x) = 10–14
 pH 7
Kw 1.4  10 5
= = 1.12 × 10–3
K a.K b 1.25  10  2
25
200  1.225  2
2 100
x 46. (A) [H+] =
 = 1.12 × 10–3 98
(0.25  x)(0.5)  x
x = [HSO4–] = 0.0114 = 1.25
800
[H ][SO24  ] [OH–] = 0.525 × × 3 = 1.26
 = Ka = 1.25 × 10 –2 1000
[HSO 4 ]
Remaining [OH–] = 10–2
pOH = 2
[H ][0.5  0.0114]
 = 1.25 × 10–2 pH = 12
[0.0114] Indicator having range 11.4 to 13.
[H+] = 2.92 × 10–4
0.8
10  0.03 (B) pH = 11 – log2 + log
0 .1
45. (A) [OH–] = = 6 × 10–4
500 = 11 + 2 log 2
= 11.6
0.08  5
[OH–] = = 8 × 10–4 Indicator having range 11.4 to 13.
500 (C) HCl m.eq = 5
Remaining [H+] = 2 × 10–4 NaOH m.eq = 13
pH = 4 – log 2 = 3.7 
0.5  10 Remaining NaOH = 8
(B) [RNH3Cl] = = 0.1 m.eq of HA = 10
50
Ha + OH–  A– + H2O
[KOH] = 0.1
10 8
RNH3Cl + KOH RNH2 + KCl + H2O
2 0 8
0.1 0.1
0.1 0.1 [A – ]
pH= PKa+log
RNH2 + H2O RNH+3 + OH– [HA]
Kb
= 5 + log 4
0.1–x x x
 5.6
Indcator having range 4.6 to 6.4

394 - Rajeev Gandhi Nagar Kota, Ph. No. 0744-2209671, 93141-87482, 93527-21564
IVRS No. 0744-2439051, 0744-2439052, 0744-2439053 www.motioniitjee.com, email-info@motioniitjee.com
Solutions Slot – 2 (Chemistry) Page # 5

EXERCISE – II MULTIPLE CORRECT (OBJECTIVE QUESTIONS)


1. (A)  V = 100 ml

pH of 10–8 m sol. of HCl is 6.97 (D) 100 M = 0.5 V

(consider the H+ from H2O also) 100 × M = 0.5 × 100

(B) M = 0.5
100 ml _________ 0.5 mole
H2PO4– – H+ HPO42–
100 ml _________ 0.055 mole of base
(C) Kw = [H+] [OH–]
On temp [H+] [OH–] wt.
0.05 = wt. = 2.25 gm
both  Kw  45
(D) HA + NaOH  NaA + H2O
C C/2 2.25
% of base = ×100
C–C/2 2.5
90%
salt
pH = pKa + log
acid
4. (A) NO as pH range is
(8.3 to 10.1 )
(B) HIn H+ + In–
2. BOH + HCl  BCl + H2O
0.1 0.9
100 m 0.5 V 20 M
100 M – 0.5 V 0 0.9
pH = PkIn + log
= 80 M 0.1
14–a PH = PkIn + 2 log 3

salt (C) Only when h <<<1


pOH = pKb + log (D) Na3PO4 + HCl  Na2HPO4 + NaCl
base
Na2HPO4 + HCl  NaH2PO4 + NaCl
20 M pH = (pk1 + pk2)/2
5 = pKb + log
80 M
 pkb = 5 – log 0.25 4. ABC

 pkb = 5.6 For alkaline solution

 Kb = 2.5 × 10–6 pH > 7

greater than 10–6


5. (A) CH3COOH/CH3COONa

3. At eq. point pH = 4.5 (B) CH3COONaHCl  CH3COOH + NaCl


pOH = 9.5 CH3COONa / CH3COOH (Buffer)

pOH = 1/2 [Pkb + pkw + hc ] (C) NH3/NH4Cl (Buffer)

9.5 = 1/2 [5.6 + 14 + hc] (D) NH3 + NaOH (not Buffer)

 C = 0.25
(C) Total volume be (100 + V)
0.25 × (100 + V) = 0.5 × C

394 - Rajeev Gandhi Nagar Kota, Ph. No. 0744-2209671, 93141-87482, 93527-21564
IVRS No. 0744-2439051, 0744-2439052, 0744-2439053 www.motioniitjee.com, email-info@motioniitjee.com
Page # 6 Solutions Slot – 2 (Chemistry)

EXERCISE – III SUBJECTIVE QUESTIONS


 pH = 11 + log 2 = 11.3

1. Kw = [H+] [OH–]
5. (a) HA H+ + A–
[H+]1 = 10–7; [H+]2 = 1.6 × 10–7
0.1 – x x x
[H+] = 0.6 × 10–7
x2
= Ka & x = 10–4.5
mole 0.1  x
2. [OH–] = 10–1 ; [H+] = 10–13
L
10 9
Number of [H+] in 1 mL  Ka = = 10–8
0.1
10  13 23 (b) pOH = 3.5
=  6.022  10
1000
x2 10 7
= Kb  = Kb
3. pH = 6.95  [H+] = 10–6.95 0.1 0.1
[H+] = (c + x) = 10–6.95  Kb = 10–6
 
from from
HCl H2O 2 C1 C1
6.  given :
 Kw = (c + x) (x) = 10–14 1 C2 C 2 = 100

10 14
 x= = 10–7.05 2
10  6.95 
1 = 10
 C = 10–6.95 – 10–7.05
= 2.3 × 10–8

1 K a1 1.8  10 5
4. (b) CH3COOH CH3COO– + H+  
7.
2 Ka2 6.2  10 10
0.1 – x x x

x2 1
 = 1.8 × 10–5 x << 0.1  x
0.1  x 
 2 = 170.4
= 1.34 × 10–3
 pH = 2.87
8. NH4OH NH4+ + OH–
0.1 2  50  0.4  1 50 C–x x x
(d) [H+] =
100 –
[OH ] = x = 10 –2.5

= 0.3
x2
 pH = 0.523  = 1.8 × 10–5
Cx
(e) Hint : Consider the [H+] from water
also. x<<C

0.1 x2 10 5
(g) Ba(OH)2 = = 10–3  C= =
100 1.8 10 5 1.8 105
 [OH–] = 2 × 10–3  C = 0.556
 pOH = 3 – log 2

394 - Rajeev Gandhi Nagar Kota, Ph. No. 0744-2209671, 93141-87482, 93527-21564
IVRS No. 0744-2439051, 0744-2439052, 0744-2439053 www.motioniitjee.com, email-info@motioniitjee.com
Solutions Slot – 2 (Chemistry) Page # 7

 x + y = 10–3 = [H+]
9. HA H+ + A– x = [CH3COO–] = 3.6 × 10–4
10–2 – x x x y = [C7H5O–2] = 6.4 × 10–4

x2 10 6 13. HA H+ + A–
Ka = =
10  2  x 10  2  10 3 0.1–x x+y x
+
Ka = 1.11 × 10–4 HB H + B–
0.1–y x+y y

K a 5.9  10 10 x ( x  y)
10.   2 × 10–10  2  10 5 ;
C 0.3 0 .1
We can neglect the 
y( x  y)
 4 10 5
x2 0 .1
= 5.9 × 10–10
Cx  (x + y)2 = 6 × 10–6
x<<C  (x + y) = 6  10 3 = [H+]
2 –10
x = 0.3 × 5.9 × 10
 pH = 2.611
x = 1.33 × 10–5 = [H+]
 pH = 4.876
14. H3PO4 H+ + H2PO –
4

0.01–x x+y+z x–y


11. CHCl2COOH CHCl2COO– + H+
H2PO4– H+ + HPO2–
4
0.01 – x x x + 0.01
y–z x+y+z z

from HCl HPO24– H+ + PO34–

x(x  0.01) y–z x+y+z z


 = 2.55 × 10–2 Ka3 << Ka2 << Ka1
0.01  x
z << y << x
 X2 + 0.0355 X – 2.55 × 10–4 = 0
 X = 6.126 × 10–3 ( x  y  z )( x  y)
 = 10–3
 [H+] = (0.01 + 6.126 × 10–3) M 0.01  x

12. CH3COOH CH3COO– + H+ x2


 = 10–3 .... (1)
0.02 – x x x+y 0.01  x
C7H5O2H C7H5O–2 + H+
( x  y  z)( y  z)
0.01 – y y x+y & = 10–8 .... (2)
xy
x(x  y) y = 10–8 = [ HPO2–]
= 1.8 × 10–5 ; (Neglect x) 4
0.02  x
(x  y  z)z
& = 10–13
(x  y)y yz
= 6.4 × 10–5
0.01  y
x.z
x (x + y) = 1.8 × 10–5 × 0.02  = 10–3 ....(3)
y
y (x + y) = 6.4 × 10–5 × 0.01
 (x + y)2 = 10–6 from equation (1)

394 - Rajeev Gandhi Nagar Kota, Ph. No. 0744-2209671, 93141-87482, 93527-21564
IVRS No. 0744-2439051, 0744-2439052, 0744-2439053 www.motioniitjee.com, email-info@motioniitjee.com
Page # 8 Solutions Slot – 2 (Chemistry)

x2 + 10–3 – 10–5 = 0
 x = 2.7 × 10–3 = [H+] x2
 = 10–2
 z = 0.37 × 10–18 0.1  x
 [PO43–] = z = 0.37 × 10–18  x2 + 0.01 x – 0.001 = 0
Rest of the problem is like problem number
Kb1 14.
15. en + H2O enH+ + OH–
0.1 – x x–y x+y 18. NiC (aq.)+H2O NiCH+ (aq) + OH– (aq.)

enH+ + H2O enH2+ + OH– 0.02–x x–y x+y


2
+ 2+
NiCH +H2O NiCH 2
(aq) + OH– (aq.)
(x – y) y x+y
x–y y x+y
x > > y due to Kb >> Kb2
1 Kb1 >> Kb2
( x  y)( x  y)  x >> y
= 8.5 × 10–5
0.1  x
x2
 x2 = 8.5 × 10–6  = 7 × 10–7
0.02
 x= 8.5  103 = [OH–]  x = 1.18 × 10–4
[OH–]  x = 1.18 × 10–4
 pOH = 3  log 8.5
 pOH = 3.928
 pH = 14 – pOH  pH = 10.072

16. H2SO4  H+ + HSO–4 19. H3A H2A– + H+


0.2
0.1–x x–y x+y+z
0 0.2 0.2 2–
H2A HA + H+
HSO4– H+ + SO2–
4
x–y y–z x+y+z
HA2– A3– + H+
0.2–x 0.2+x x
y–z z x+y+z
(x  0.2)x x >> y >> z
 10–2 =
0.2  x
x2
 2
x + 0.21 x – 0.02 = 0  = 10–5  x = 10–3
0.1
Solve the equation and find out the value
of x.
y.x
[H+] = 0.2+x ; [SO 2–]=x  = 10–9  y = 10–9
4
x
[HSO4–] = 0.2 – x ; H2SO4 = 0

z.x
 = 10–13  z = 10–19
17. H2C2O4 HC2O–4 + H+ y
0.1–x x–y x+y
20. NH3/NH4+ : Basic Buffer
– 2– +
HC2O C2 O + H
4 4 NH3 + H2O NH4+ + OH–
x–y y x+y 0.05–x x+C x
x >> y because of K1 >> K2 NH4Cl NH4+ + Cl–

( x  y)( x  y) C X O
 = 10–2 O C+X C
0.1  x

394 - Rajeev Gandhi Nagar Kota, Ph. No. 0744-2209671, 93141-87482, 93527-21564
IVRS No. 0744-2439051, 0744-2439052, 0744-2439053 www.motioniitjee.com, email-info@motioniitjee.com
Solutions Slot – 2 (Chemistry) Page # 9

X + C = 0.1 x << 0.2


 x2 = 3 × 10–10
0.1X
 = 1.8 × 10–5  x = 1.7 × 10–5 = [OH–]
0.05  X  pOH = 4.76
 [OH–] = X = 9 × 10–6 (B)
Equilibrium shift backward
21. CH3COOH + NaOH  CH3COONa + H2O  [OH–] 
m.eq. 10 5  pOH  and pH 
5 0 5 (C)
Acidic Buffer Basic buffer

[CH 3COONa ] salt


pH = pKa  log pOH = pK b  log
[CH 3COOH ] base
 pH = pKa + log 1
0 .3
 pH = 5 – log 1.8  pOH = 9  log 1.5  log
0.1
 pH = 4.74
pOH = 9.301
22. (NH4)2 SO4 = x  NH4+ = 2x  pH = 14 – 9.301
NH3 = 0.1  pH = 4.699

2x 25. m.eq. NH3 = 20


pOH = pKb + log
0 .1 m.eq. H+ = 15
m.eq. of NH+4 formed = 15
2x
 4.74 = 4.74 + log m.eq. of NH3 remaining = 5
0 .1 pOH = 5 – log 1.8 + log 3
 x = 0.05 = 5.222
 pH = 8.778
+ –
23. NH4Cl  NH 4
+ Cl
m.eq. 7.5 7.5 7.5 26. NH+4 + OH–  NH4OH
NH4+ + OH–  NH4OH 10 7.5
7.5 5 2.5 0 7.5
2.5 0 5 Basic buffer
pKb = 14 – 9.26 = 4.74
2 .5
 pOH = 5  log 1.8  log
2 .5 7 .5
pOH = 4.74  log
5 = 4.2676
 pOH = 4.44  pH = 9.7324
 pH = 14 – 4.44
 pH = 9.56 27. HIn H+ + In–+
c/2 c/2 c/2
24. (A) Ka = c/2 = 10 –2

+ –
C5H5N + H2O C5H5NH + OH Now
0.2–x x x HIn H+ + In–
c’–c’ c’ c’
x2
= 1.5 × 10–9
0.2  x

394 - Rajeev Gandhi Nagar Kota, Ph. No. 0744-2209671, 93141-87482, 93527-21564
IVRS No. 0744-2439051, 0744-2439052, 0744-2439053 www.motioniitjee.com, email-info@motioniitjee.com
Page # 10 Solutions Slot – 2 (Chemistry)

(4 10 3 ) 2 x2
10–2 = = 6 × 10–5
c'4  10  2 cx
 c1 = 5.6 × 10–2
1010
3  = 6 × 10–5
4  10 c  10 5
 =
5.6  10  2
 (1 – ) % = 28.57% 7  10 5
 c=
6
28. HIn H+ + In–
x 6
c–c x x  =
c 7
x2
= 4 × 10–4 x 6
cx % =  100
c 7
[H  ][In  ] = 85.7%
KHIn =
[HIn]
31. HInOH H+ + OHIn–
Ionised Red Blue
pH = pKHIn + log
Unionised c–x x x
75% Red
pHmax = pKHIn = 4 – log4
Acidic medium cx
 100 = 75
(a) Basic × c
(b) Acidic 
(c) HCI + NaOH  c
 x=
 4
Depend on concentration
(c 2 / 4 2 )
 3  10 5 
29. H2BO–3 + H+ H3BO3
3c / 4
 c = 36 × 10–5
 x = 9 × 10–5
0.1/2 0.1/2
 [H+] = 9 × 10–5
y y 0.1/2
 pH = 5 – log9
10 10 0.1 75% Blue
 =
7.2 2y2  x = 3c/4

 y = 6 × 106 = [H+] 4
 c=  105
 pH = 5.22 3
Methyl orange  x = [H+] = 10–5
pH organe (4.2 to 6.3)  pH = 5
pH = log 9
30. HIn H+ + In–  pH = 0.954
c–x x x

394 - Rajeev Gandhi Nagar Kota, Ph. No. 0744-2209671, 93141-87482, 93527-21564
IVRS No. 0744-2439051, 0744-2439052, 0744-2439053 www.motioniitjee.com, email-info@motioniitjee.com
Solutions Slot – 2 (Chemistry) Page # 11

[H+] = 2 × 10–3 = x
32. CH3OO– + H2O Kw CH3COOH +OH–
Ka
K w  0.25
 Kb =
0.08–x x x+c x2

from water 10 14  0.25
=
Kw 10–9 x(x  c) 4  10  6
 
Ka 1.8 0.08  Kb = 6.25 × 10–10

0.08 35. Weak acid + Weak base : Salt


(x+c)x=  10– 9
1 .8 Hydrolysis
and
Kw Ch.Ch
(x+c) c=10–14 = 2
K a .K b C (1  h 2 )
8
(x+c)2=  10–11  10–14
1 .8
Kw
 h=
80 K a .K b
(x+c)2=  10 12
1 .8
 h = 5.56 × 10–3
(x+c) = [OH–] = 6.66×10–6
% hydrolysis = 0.556 %
K w .K a
33. NH+4 + H2O NH4OH + H+ [H+] = ; Ka = Kb
Kb

2–x x x = K w = 10–7
 pH = 7
Kw 10 9 x2
= =
Kb 1.8 2x
36. CN– + H2O HCN + OH–
Solve for x and find out pH
or
0.06–x x x
1
pOH = [pKw + pKb + log c]
2 Kn= kw/Ka  10 14
6  10–10
1
=
2
[14 + 5 – log 1.8 + log 2]
x2 1 4
Kh = =  10
= 9.523 0.06  x 6
 pH = 4.477  x = 10–3

10 3
34. C5H6N+ + H2O C5H5N + H3O+ % Hydrolysis =  100
0.25–x x x
0.06
= 1.667 %
Kw x2 x2
= = (neglect x)
Kb 0.25  x 0.25
37. CrO 2–
4
+ H2O HCrO4– + OH–
 Given pH = 2.699
 log [H+] = – 2.699
0.005–x x x

394 - Rajeev Gandhi Nagar Kota, Ph. No. 0744-2209671, 93141-87482, 93527-21564
IVRS No. 0744-2439051, 0744-2439052, 0744-2439053 www.motioniitjee.com, email-info@motioniitjee.com
Page # 12 Solutions Slot – 2 (Chemistry)

Further hydrolysis of HCrO4– is not possible

x2 10 14 40. C6H5O– + H2O C6H5OH + OH–


 =
0.005  x 3.1 10 7 10–3–x x x
 x = 1.3 × 10–5
x2
 Extent of hydrolysis = 10–4
10 3  x
1.3  10 5  x2 + 10–4 x – 10–7 = 0
=  100
0.005  x = 2.7 × 10–4
= 2.6 × 10–1 %  pOH = 4 – log 2.7
= 0.26 %  pH = 10 + log 2.7
 pH = 10.43
38. Same as question number 36.
1
41. pH = [ pK a1  pK a 2 ]
2
39. PuO2+2 + H2O PuO2OH+ + H+
1
0.01–x x–y x+y = [7 – log 4.5 + 11 – log 4.5]
2
PuO2OH+ + H2O PuO2(OH)2 + H+
1
= [18 – 2 × log 4.5]
2
x–y y x+y
= 8.346
Kw
x + y = 10–4 ; K b2
1
42. pH = [ pK a1  pK a 2 ]
2
( x  y)( x  y)
=
0.01  x 1
= [2.94 + 5.44]
Kb1 >> Kb2 2
 x >> y = 4.19

Kw y.x
 43. (A)
K b1 = x
Zn2+ + H2O Zn(OH)+ + H+

10 14 0.001–x x x
 y=
K b1
x2 x2
10–9 = =
Given 0.001  x 0.001
[H+] = x = 10–4  x2 = 10–12
–8
 Kb2 = 10 [H+] = 10–12
[H+] = x – 10–6
Kw
and Kh = K = 10–6 (B) Zn(OH)+ Zn2+ + OH–
b2

( Zn 2  )(OH  )
Kb =
[ Zn(OH)  ]

394 - Rajeev Gandhi Nagar Kota, Ph. No. 0744-2209671, 93141-87482, 93527-21564
IVRS No. 0744-2439051, 0744-2439052, 0744-2439053 www.motioniitjee.com, email-info@motioniitjee.com
Solutions Slot – 2 (Chemistry) Page # 13

HX + NaOH  NaX + H2O


[H  ][OH  ][Zn 2  ] Let the molarity of NaOH be M and m.
Kb =
[H  ][Zn(OH)  ] moles of HX = C
Hx + NaOH  Nax+H2O
C–10M 10M–10M 10 M
Kw 10 14 C–20M 20M–20M 20 M
= =
Ka 10 9 10M
K
5.8 = P a  log .....(1)
 Kb = 10 –5 C – 10M

K 20M
44. CH3COOH + NaOH  CH3COONa +H2O 6.402 = P a  log .....(2)
C – 20M
From eq (1) (2)
C=30 M
 PK a =5.8+log 2 = 6.1
48. Weak acid + Strong acid
At equivalent point
M1 V 1 = M2 V 2
At equlvalent point ; m.eq. of both are 40 M = 35 × 0.1
equal.  M = 0.0875 for Acid
m.eq. of CH3COONa formed = 0.1 × V pH = 5.75 < 7. Buffer solution
HX + NaOH  NaX + H2O
0. 1  V 3.5 2
[CH3COO–] = = 0.05 1.5 0 2
2V
 5.75 = pKa + log 2/1.5
CH3COO– + H2O CH3COOH + OH–  pKa = 5.625
&
1 Ka = 2.37 × 10–6
pH = [14 + pKa + log C]
2
49. Same as question 47.
Hint : Buffer solution.
1
= [14 + 5 – log 1.9 + log 0.05]
2 50. Let molarity of acid = M
find out pOH = 14 – pH & HX + NaOH  NaX + H2O
(i) 50 M 1
[OH–] concentration.
50 M–1 0 1

45. Same as 44th problem. (ii) 50M 2.5


50M–2.5 0 2.5
46. [Salt] = 0.2 1
4.16 = pKa + log ... (1)
1 50M  1
pOH = [14 + pKb + log C]
2 2 .5
4.76 = pKa + log ... (2)
50M  2.5
1 Solve the equation (1) and (2) and find
= [19 – log 1.8 + log 0.2]
2 out the volume of Ka.
 pH = 14 – pOH = 4.98
52. MX M+ + X–
S1 S2
47. Weak acid + strong base
OX2 Q2+ + 2X–

S1 2S2
pH < 7 (Acidic)
S12 = 4S32
Only possibility is for Buffer solution.

394 - Rajeev Gandhi Nagar Kota, Ph. No. 0744-2209671, 93141-87482, 93527-21564
IVRS No. 0744-2439051, 0744-2439052, 0744-2439053 www.motioniitjee.com, email-info@motioniitjee.com
Page # 14 Solutions Slot – 2 (Chemistry)

Ksp = 108 S5 = 1.1 × 10–23


S12  S = 10–5
=4
S32
59. AgCl Ag+ + Cl–
S12 = 4 × 10–18
 S1 = 2 × 10–9 S S + 0.2
4S32 = 4 × 10–18 
 S2 = 10–6 from BaCl2
S2 > S1 S (S + 0.2) = 10–10
QX2 is more soluble S << 0.2
 S = 5 × 10–10

Mole 60. Pb(NO3)2+2NaCl  PbCl2(s) + 2NaNO3


53. First change the value in then cal-
L 10 m.mole 75 m.mole
culate the Ksp. 10
(Pb2+) = = 0.1 ; Cl = 0.75
+ –
100
54. CuI Cu + I
Qsp = (Pb2+) (Cl–)2 = 0.225
S (0.1 + S) Qsp > Ssp ppt formation
NaI  Na+ + I–
0.1 0.1
S (0.1 + S) = 5 × 10–12 Pb2+ + 2Cl– PbCl2 (s)
S << 0.1
0.1 0.75
Mole y 0.55 0.1
 S = 5 × 10–11
L 1 1
 =
K sp y(0.55) 2
55. CaF2 Ca2+ + 2F–
S 2S  y = 5.26 × 10–4

Ksp = 4S3 : given 2S = 4.1 × 10–4 61.


y = 5.62 × 10–4 Mole/L
3
4
 4  10 
 Ksp = 4  = 3.45 × 10–11 5.62  104  100
 2
2+
 Mole of Pb left =
1000
= 5.62 × 10–5
2+ –
56. ML2 M + 2L Mass of Pb2+ left = 5.62 × 10–5 × 207.2
S 2S = 0.01164 g
= 11.64 mg.
2.4  10 5
S=  10 Mole/L
60 62. Mg(OH)2 Mg2+ + 2OH–
(a)
Ksp = 4S3 = 2.56×10–16
Qsp = (10–5)2 (10–3) = 10–13
57. pH = 8 Qsp < Ksp : No ppt
 pOH = 6 & [OH–] = 10–6 (b) Qsp = (10–3)2 (10–3) = 10–9
Fe(OH)3 e3+ + 3OH–
Fe Qsp > Ksp : ppt will form
S 10–6
–6 3
S (10 ) = 1 × 10 –36 63. PbI2  PbI2
(Solid) (aq.)
Mole S = solubility
 S = 10–18
L PBI2 Pb2+ + 2I–
S × 0.9 2S × 0.9
58. A 2 X3 2A3+ + 3X2– Ksp = (S × 0.9) (2S × 0.9)2
2S 3S  4S3 × (0.9)3 = 1.4 × 10–8

394 - Rajeev Gandhi Nagar Kota, Ph. No. 0744-2209671, 93141-87482, 93527-21564
IVRS No. 0744-2439051, 0744-2439052, 0744-2439053 www.motioniitjee.com, email-info@motioniitjee.com
Solutions Slot – 2 (Chemistry) Page # 15

 S3 = 4.8 × 10–9 Assume all dissolve


 S = 1.68 × 10–3 Ag+ + 2Cl– AgCl2–
0.01 0.01+C
64. Qsp > Ksp for AgCl 
(5 × 10–3) (Ag+) > 10–10 added by NaCl
(Ag+) > 2 × 10–8 y 0.01 + C – 0.02 0.01
(Ag+) (Br–) = Ksp = 5 × 10–13 y C – 0.01 0.01
(Br–) < 2.5 × 10–5 Ksp = 10–10 = y (C – 0.01) ... (1)
pH = 3 &
 [H+] = 10–3
AgCN Ag+ + CN– 0.01
3 × 105 = ... (2)
S Y y(C  0.01) 2
CN– + H+ 
1 HCN
Ka 103
 C= mole in 100 L (from que.)
S 10–3 3
–3
Y 10 – S S
103
1 S  Mass of NaCl =  58.5 g = 19.5 kg
= 3
Ka Y  1 0 3
 S << 10–3 68. Fe3+ + SCN– Fe(SCN)2+
1 S Fe(SCN)2+ + SCN– Fe(SCN)2+
 = ... (1)
Ka Y  10 3 Fe(SCN)+2 + SCN– Fe(SCN)3
____________________________
&
S.Y = Ksp ... (2) Fe3+ + 3SCN– Fe(SCN)3
Kf = K1 K2 K3
65. AgSCN Ag+ + SCN– = 2080
S1+S2 S1
AgBr Ag+ + Br –
1 1
Kdis. = =
S1+S2 S2 Kf 2080
S1 (S1 + S2) = 1.1 × 10–12 Kdis. = 4.8 × 10–4
S2 (S1 + S2) = 5 × 10–13
 (S1 + S2)2 = 16 × 10–13
 S1 + S2 = 1.26 × 10–6 69. AgBr Ag+ + Br–
 S1 = 8.7 × 10–7
S S
&
y S
S2 = 3.97 × 10–7
Ag+ + 2NH3 Ag (NH3)2+
66. MgF2 Mg2+ + 2F– S 0.4
y 0.4 – 2S S
S1 2S1+2S2
y. S = 5 × 10–13
SrF2 Sr2+ + 2F– and
S2 2S2 + 2S1
4S1 (S1 + S2)2 = 9.5 × 10–9 S
= 108
& y(0.4  2S) 2
4S2 (S1 + S2)2 = 4 × 10–9
2S << 0.4
 4(S1 + S2)3 = 13.5 × 10–9
 S1 + S2 = 1.5 × 10–3 S
[F–] = 2S1 + 2S2  = 108
y(0.4) 2
= 3 × 10–3 M
 S2 = 0.8 × 10–5
67. AgCl Ag+ + Cl–  S = 2.8 × 10–3 M.
0.01 0.01

394 - Rajeev Gandhi Nagar Kota, Ph. No. 0744-2209671, 93141-87482, 93527-21564
IVRS No. 0744-2439051, 0744-2439052, 0744-2439053 www.motioniitjee.com, email-info@motioniitjee.com
Page # 16 Solutions Slot – 2 (Chemistry)

EXERCISE – IV ADVANCED SUBJECTIVE QUESTIONS


Sol.1 [H3O+] Concentration neglecting the ionization of
H2O = 10–8 M x x
Now, Considering ionization of water v

v = x2
Also,
H2O, H+ + OH–
x (10 + x)
1
1  x  v 1  x 
–6
v
We have, \kw = x(10–6 + x) = 10–14
x = 0.99 × 10–8 M x
  2  1.8  10 5
v
108  0.99  108 x = 1.8 × 10–5 V
% error =  100 = 1 %
10 8
x2
Sol.2 HCl H+ +
Cl–   1.8  10 5
0.09 – 0.09 0.090.09 v 1  x 
Cl2HCCOOH Cl2COO– + H+ 1.8 × 10–5 V = 1 – 1.8 × 10–5 V
0.1 – y y y + 0.09 + x 3.6 × 10–5 V = 1
We have, y + 0.09 + x = 10–1 = 0.10 1
x + y = 0.01 V = = 2.77 × 104 litre
3.6  105
0.1 y 0.01
Also, K CH3cOOH  10 5 = 
0.01  y 0.1 Sol.6 (a) Concentration of CH3NH2
 x 0.01 PV 1 959
C  = 39.25 M
x  0.1 RT 0.082  298
 K Cl2HCOOH 
0.09  0.01
1
0.01 0.1 POH = [PKb– logC] = 0.898  0.13 M
 1.25 × 10–2 2
0.08  Concentration of NaOH needed = 0.13 M
(b) –log[OH–] = 0.0898
Sol.3 MW of ClCH2COOH = 44.5 (OH–) ~ 0.13 M
 Concentration of NaOH needed = 0.13 M
9.45
No. of Moles of acid = = 0.1
94.5 Sol.7 2Ag(s) + 2I– + 2H2O 2Ag(s) + H2(g) + 2OH–
 [acid] = 0.1 × 2 = 0.2 M
ClCH2COOH ClCH2COO– + H+ [H2 ][OH ]2
K eq  ..............(1)
0.2(1 – ) 0.1  10–2 = 0.2  [I ]2
10 2  105 For H2(s)
 = PV = nRT,
2
P = CRT
= 0.05 P 0.6
C=  = 0.0245
RT 0.082  298
K w 50ºC H  1 1  From eqn(1)
Sol.4 log K  =   323  298 
w 25 ºC 2 . 303   1.2  10 2  0.12
[OH–]2 =
5.474 H 25 0.0245
 log   [OH–] = 0.0699
1.08 2.303  8.314 323  290
(H)ionization of H2O = 51.963 KJ/mol 1.2  10 2  0.12
 (H)Neutralization = – 51.963 KJ/mol [OH–]2 =
0.6
= 1.650
Sol.5 CH3COOH CH3COO– + H+
1–   Sol.8 (a) H3PO4 + NaOH NaH2PO4 + H2O
 2 Initially
K =    Ka  1.8  10 5 milli
1 
no. of moles 6 3 0
After 3 0 3
PH = – log(H+) = -log 1.8  105 = 2.37

394 - Rajeev Gandhi Nagar Kota, Ph. No. 0744-2209671, 93141-87482, 93527-21564
IVRS No. 0744-2439051, 0744-2439052, 0744-2439053 www.motioniitjee.com, email-info@motioniitjee.com
Solutions Slot – 2 (Chemistry) Page # 17

 Formation of Acidic Buffer of H3PO4 and


2
NaH2PO4 takes place = 12 + log = 12
2
[salt]
PH = Pk1  log
[ Acid]
Sol.9 Na2CO3 + HCl  NaHCO3 + NaCl
3 Initially 2 2 0
= 3 – log 7.5 + log
3 no. of milli
= 2.12 moles
(b) H3PO4 + NaOH  NaH2PO4 + H2O After R × n 0 0 2
Initially Now, NaHCO3 is an amphiprotic salt
no. of
1
milli moles 6 6 0  PH =
2

( Pk1  Pk 2 = 8.34 
After R × n 0 0 6
We need to calculate P H of amphiprotic salt (b) Na3PO4 + HCl 
NaH2PO4 Na2HPO4 + NaCl
Initially 0.8 1.6
1 0
PH = (Pk1  Pk 2 )
2 no. of milli
= 4.66 moles
(c) H3PO4 + NaOH  NaH2PO4 + H2O After R × n 0 0.8 0.8
Initially
No. of (c) Na3PO4 + NaH2PO4 
milli moles 0 2.4 4.8 Na2HPO4 + NaCl
After R×n Initially 5 5
0
NaH2PO4 + NaOH  Na 2HPO 4 no. of milli
+ H2 O moles
4.8 – 2.4 2.4 – 2.4 2.4 After R×n 0 0
= 2.4 =0 = 10
2.4
1
 The mixture is buffer of NaH 2PO 4 and  PH =
2 2

Pk  Pk 3 = 9.6 
Na2H2PO4 and Na2HPO2
(d) H3PO4 + Na3PO4 
[Na2HPO 4 ] Na2HPO4 + NaH2PO4
 PH = Pk 2 + log [NaH PO ]
2 4 Initially 4 4
0 0
2.4 no. of milli
= 8 – log 6.2 + log
2.4 moles
= 7.2 After R×n 0 0 4
(d) H3PO4 + NaOH  NaHPO4 + H2O 4
Initially 4 The mixture is acidic buffer solution of Na2HPO4
10 0 and NaH2PO4
no. of milli
moles [H2PO 4 ]
 PH = Pk 2  log
After R × n 0 6 [HPO 22 ]
6
= 7.2
Na2HPO4 + NaOH  Na2PO4 + H2O
4–4 6–4 4
Sol.11 The acid should be CH3COOH
=0 =2 =4
Na2HPO4 + NaOH  Na3PO4 + H2O [Salt]
4–2 2–2 2 & pH = Pk a  log
[ Acid]
=2 =0 =2
[Salt ]
log = 5 – 4.74 = 0.26
 The mixture is buffer of Na 2HPO 4 and [ Acid]
Na3PO4
[Salt] 1.8
[Na3PO 4 ] =
 PH = Pk 3  log [Na HPO ] [ Acid] 1
2 4
[ Acid] 1

[Salt ] 1.8

394 - Rajeev Gandhi Nagar Kota, Ph. No. 0744-2209671, 93141-87482, 93527-21564
IVRS No. 0744-2439051, 0744-2439052, 0744-2439053 www.motioniitjee.com, email-info@motioniitjee.com
Page # 18 Solutions Slot – 2 (Chemistry)

Concentration (0.05 –x)/10 x/10


Sol.12 (i) PH of 0.1 M NaHCO3 (x+0.06)/10
HCOONa  HCOO–
1
PH = [PK1 + PK2] + Na+
2 0.06 – 0.06 0.06 + x
= 8.35
( x  0.05) / 10  x / 10
1 Ka =
(ii) PH of Na2HPO4 = [PK2 + PK3] = 9.6 (0.05  x ) / 10
2
( x  0.06)x
1  1.8  10 4
(iii) PH of NaH2PO4 = [PK2 + PK3] = 4.66 (0.05  x )  10
2
 x 
Sol.13 BOH + HCl  pH = – log[H+] = – log   = 3.85
BCl + H2 O  10 
Initially 4 0.16 ×V x  0.06  x
0 (c) Ka =
(0.05  x )  100
no. of milli
moles  x 
After R×n 0 0 pH = – log[H+] = – log   = 3.99
4  100 
 0.16 × V = 4 Sol.15 NH4Cl  NH4+ + Cl–
NH3 + H2O  NH4+ + OH–
4
 Concentration of BCl =
25  40 [NH4 ]
POH = PK b  log
4 [NH3 ]
= 6.15 × 10–2 M = 5 –log 1.8 + log(1)
65
= 4.74
1 Now,
 pH =[pk 2  pk b  log C] = 5.23
2
[Salt  x ]
 pkb = 4.75 P'OH  PK b  log
Now, [Base  x]
BCl + NaOH  where x is moles of NaOH added
BOH + NaCl  P'OH PK b = P'OH POH = 1
Initially 4 1.8 0
0 [Base  x ]
no. of milli = log
[Salt  x ]
moles
After R×n 4–1.8 0 1.8 0.1  x
  10
1.8 0.1  x
= 2,2 =0  0.1 + x = 1 – 10x
=1.8  11 x = 0.9
The mixture is buffer of BOH and BCl  x = 0.0818 moles
[BCl]
 POH = pkb + log Sol.16 CH3COOH + NaOH  CH3COONa + H2O
[BOH]
Initially
2.2 no. of milli moles 2 0.1
= 4.75 + log 0
1.8
After r × n 1.90 0
POH = 9.16
0.1
[Salt]
[HCOO ] (PH )1 = PKa + log
Sol.14 (a) PH = Pka + log [ Acid]
[HCOOH]
0.1
0.06 = PKa + log
= 4 – log 1.8 + log = 3.82 1.9
0.05
(b) HCOOH  H+ + 1.9
Similarly (PH )2 = PKa + log
HCOO– 0.1
No. of moles0.05 – x x
x + 0.06

394 - Rajeev Gandhi Nagar Kota, Ph. No. 0744-2209671, 93141-87482, 93527-21564
IVRS No. 0744-2439051, 0744-2439052, 0744-2439053 www.motioniitjee.com, email-info@motioniitjee.com
Solutions Slot – 2 (Chemistry) Page # 19

2 ×10–12
(1.9)2 H2PO4– + H2O H3O+ + HPO42– (As Acid)
= pH = (PH)2 – (PH) = log 2 = 2.558
(0.1) K1 = 5 ×10–3
K1 >> K11
Sol.17 Na3PO4  3Na+ + PO43– Hence solution of H2PO4 with H2O is acidic.

PO43– + H2 O HPO42– + OH–


HPO42– + H2O H2PO4– + OH– (As base )
K 10 14
K '3  w  Kw 10 14
K 3 4.5  10 13 K21 = K =
2 5  10 8
0.1 (1–1) 0.1 1(1–)0.11 + 0.1 12 + 0.1
123 = 2 × 10–7

HPO43– + H2 O H2PO42– + H2PO42– + H2O H2PO4– + OH– (As base)

Kw 10 14 Kw 10 14
K '2   K '2   =
OH –
K 2 6.3  10 8 K 2 5  10 8
0.1 (1–2) 0.1 12(1–)0.112 + 0.1 1 + 0.1 2 ×10–7
123 H2PO42– + H2O PO42– + H3O+ (As Acid)
K2 = 5 × 10–8
H2PO4– + H2 O H3PO4 + Now, K’2 > K2 , Solution worked as base
PO43– + H2O HPO42– + OH– (As base)
14
Kw 10
K1'   Kw 10 14
OH– K1 7.1 10 3 K '3   = 2 × 10–2
K 3 5  10 13
0.1 12(1–3) 0.1 123 0.1123 + 0.1 1 +
0.1 12
Now 1 – 2 1 [NaHCO3 ]
1 – 3 1 Sol.19 PH = Pk1  log
[H2CO3 ]
or [OH–] ~ 0.1 1
We have 0.0005
 8 = 7 – log 5 + log [H CO ]
[OH ][HPO 24 ] 0.11  0.11 2 3
K '3  = =
[PO 34 ] 0.1(1  1) 0.0005
 log [H CO ] = 1 + log 5 = 1.698
0.022 2 3
0.1 12 = 0.022 – 0.022 1
0.1 12 + 0.022 1 – 0.022 = 0 0.0005
 [H CO ] = 101.698 = 49.89
1 = 3.73 × 10–2 M 2 3
 [H2CO3] = 10–5
[OH–] = 0.1 3.73 ×10–2 M H2CO3 H+ + HCO3–
[OH ][H2PO 4 ] 10–5
10–8 [HCO3–]
K '2 
[HPO 24 ] [H ][HCO3 ] 10 8 [HCO3 ]
We have K1 = = =5×
As, [OH—] [HPO42–] [H2CO3 ] 10 5
10–7
We have, [H2PO4–] = K21 = 1.587 × 10–7 M  [HCO3–] = 5 × 10–4 M
Also,
' [OH ][H3PO 4 ]
So, K1  HCO3– CO32– + H+
[H2PO 4 ] 5 × 10 –4
[CO32–] 10–8

1014 1.587  10 7 [CO32 ][10 8 ]


[H3PO4] =  = 6 ×10–18 M K2 = = 5 × 10–13
7.1 10 3
3.73  10 2 5  10 4
 [CO32–] = 2.5 × 10–8 M

Sol.18 H2PO4– + H2O H3PO4 + OH– (As Sol.20 Fe3+ + H2 O Fe(OH)2+ +


+
base ) H
1 0.05 0.05 0.05
Kw 10 14 conc at eqn
K1'   = V V V
K1 5  10 3

394 - Rajeev Gandhi Nagar Kota, Ph. No. 0744-2209671, 93141-87482, 93527-21564
IVRS No. 0744-2439051, 0744-2439052, 0744-2439053 www.motioniitjee.com, email-info@motioniitjee.com
Page # 20 Solutions Slot – 2 (Chemistry)

0.05 1
(PH)1 for th neutralisation
V 4
0.05 / V 2 4
we have Keq = 6.5 × 10–3 = (PH)1 = PKa + log
1  0.05 / V  43
100 4
 V= (PH)2 for th neutralisation
19  13 3
0.05 34
 (H+) = = 0.1235 (PH)2 = PKa + log
V 4 1
PH = – log(H+) = 0.908
1
(PH)2 – (PH)1 = log 3 – log
3
Dissociated form = log 3 + log 3 = 2 log 3 = 0.9542
Sol.21 PH = K2x + log
Undissociated form
1
 Kin = 7.2 Also, at rd neutralisation
3
In first case,
3
5 PH = PKa + log
 PH = 7.2 + log = 7.9 32
1
PKa = 4.45 + log 2 = 4.751
In second case
7.9 = (Kin)2 + log 4 1 10
Between th & th stages of
(Kin)2 = 7.9 – log 4 = 7.3 11 11
Also , neutralisation the PH change by 2 and 8 .
PH = 7.3 + log I = 7.3
Sol.25 Mg(OH)2 Mg+2 + 2OH–
Sol.23 BOH + HCl  BCl + H2O Conc. 10 –4
10–5

a a As, Ionic product, 10–4 × (10–5)2 < Ksp


a 6 No ppt occurs.
4 4
Min value of [OH–] for pptn to occur
3 a 10–4 × [OH–]2 = Ksp
= 24 × = 18 meq Now, = 6 m eq
4 4
[OH–] = 10 7
[salt ]
POH + log a = 24 m eq 10 14
[basic ] +
[H ] =
4 [OH ]
= PKb + log = 14 -9.24 = 4.76 PH = – log[H+] = 10.5
43
 PKb = 5.237
Now, BCl + NaOH  BOH + NaCl Sol.26 Zn(OH)2 Zn2+ + 2OH–
6–6 6–6 6 6 1-x-y x [OH ]

=0 =0 =6 =6 Zn(OH)2 + 2OH– [Zn(OH)4]2–


Total BOH = 6 + 18 = 24 m eq 1-x-y [OH ]–
y
we have
24 Ksp x[OH–]2 = 1.2 × 10–17
(BOH) = M
50
1.2  10 17
1 x=
POH = [Pkb – logC] [OH ]2
2
1 24 y
= [5.237 – log ] = 2.774 Also, KC = 0.13 =
2 50 [OH ]2
 PH = 11.22  y = 0.13 [OH–]2

Sol.24 HA + NaOH  NaA + H2O 1.2  10 17


Total solubility = x + y = + 0.13 [OH–
1 1 [OH ]2
1
4 4 ]2
For, min or many
3 1
= =
4 4

394 - Rajeev Gandhi Nagar Kota, Ph. No. 0744-2209671, 93141-87482, 93527-21564
IVRS No. 0744-2439051, 0744-2439052, 0744-2439053 www.motioniitjee.com, email-info@motioniitjee.com
Solutions Slot – 2 (Chemistry) Page # 21

Sol.29 AgNO3 + HCN  AgCN + HNO3


d( x  y )
0
d[OH ] 0.02
0 0
2
 2  1.2  10 17 0.02
 + 0.26 [OH–]= 0 = 0.01
[OH ]3 2
AgCN Ag++ CN–
2  1.2  10 17 x x–y
 (OH–)4 =
0.26 CN– + H2O HCN + OH–
1/ 4 x–y y y
 2  1.2 10 16  We have, Ksp(AgCN) = x (x – y) = 4 × 10–16
 (OH ) –
=   
 2.6 10 
y2 1014
Also, =
POH = 4 × log 0.98 = 4.0086 xy 9  1010
PH = 9.99 AgNO3 Ag+ + NO3–
and solubility = x + y = 10–9 M 0.01 0.01
HCN H+ + CN–
Sol.27 If x be the concentration of AgCl in the so- 0.01 - x x x–y
lution, then [Cl–] = x Ag+ + CN– AgCN
From the Ksp for AgCl, we derive 0.01 - y x–y y
K sp (0.01-y) (x-y)= Ksp(AgCN) = 4 × 106
1.7  10 10
[Ag+] = =
[Cl ] 
x x( x  y )
= 9 × 10–10
If we answer that the majority of the dissolved Ag+ (0.01  x )
goes into solution as Ag(NH3)2+ then
(0.01  y )  0.01 4  10 6
Ag(NH3)2+ then 
Ag(NH3)2+ = x
 x 9  10 10
Since two molecules of NH3 are required for ev-
ery Ag(NH3)2+ ion formed, we have 4
 (0.01 – y ) =  10 4  10 2 x
[NH3] = 0.2 – 2x 9

[ Ag ][NH3 ]2 Sol.30 MA M+2 + A2–


 K inst = =
[ Ag(NH3 )2 ] 1-& & & –x
H2 O + A 2– Kw/K2 HA– + OH–
 1.7  10 10 
 0.2  2x 2 x–x x–y x+y

 x 
 we now ,
x
K w [H ][OH] [HA  ][OH ]
= 6 × 10–8   =
 x = [Ag(NH3)2+] = 9.6 × 10–3 M, which is the K2 K2 [ A 2 ]
solubility of AgCl in 0.2 M NH3
( x  y )( x  y )
..........(1)
Sol.28 Let x be the concentration of Ag in the 0.05 + Sx
M solution of Ag(CN)–2. Then
K w [H ][OH ] [H2 A ][OH ]
Ag(CN)2– Ag+ + 2 CN–   =
0.05 – x x 2x K1 K1 [HA  ]

[ Ag ][CN ]2 y( x  y )
Kinst = = 4 × 10–19 ..........(2)
[ Ag(CN)2 ] (x  y)
Ksp = s(s – x) ..............(iii)
x( 2x )2 4x3 from (i), (ii), & (iii)
=  = 4 × 10–19
(0.05  x ) 0.05 1/ 2
  2 2 
 x = 1.7 × 10–7 M
S =  sp 

K 1  [H ]  H   
Ionic product of AgCl in the solution = (1.7 × 10–  K 2 K1K 2 
7
)0.02 = 3.42 × 10–9 M2   
Since the value exceeds the solubility product of
AgCl, therefore pptn will be.

394 - Rajeev Gandhi Nagar Kota, Ph. No. 0744-2209671, 93141-87482, 93527-21564
IVRS No. 0744-2439051, 0744-2439052, 0744-2439053 www.motioniitjee.com, email-info@motioniitjee.com
Page # 22 Solutions Slot – 2 (Chemistry)

EXERCISE – V JEE-PROBLEMS
x = 10–5
1. HCl + NaOH  NaCl
200×10–2 300×10–2 10 5
% HOH =  100
1 0.1
[OH] = = 10–24
500
2 × 10–3 = 0.01 %

2. Ph(OH)2 Pb2 + 2OH– 0.1  V


Ksp = 4s3 10. [A–] = = 0.05
2V
= 4s3
= 4 × (6.7 × 10–6) 1
pH = pKa  pKw  log C
= 1203.52 × 10–18 2
1.2 × 10–15 pOH = 3.9
1.2 × 10–15 = S1 × 10–15 pH = 10.10
S1 = 1.2 × 10–3 [H+] = 7.9 × 10–11

12. Ag+ NH3  Ag


6.31
4. No. of H2C2O4 2mv = ky = K1 × K2
126 = 3.5 × 10–3 × 1.7 × 10–3
= 0.05 = 5.95 × 10–6
0.05
[H2C2O42mv] = = 0.2 14. S2 = 4.0 ×10–8
0.25
S = 2 × 10–4
2 × 10 × 0.2 = 0.1 × V 4sn = 3.2 × 10–14
V = 40 ml 4s3 = 32 × 10–15
s = 2 × 10–5
5. CH3COOH CH3COO– + H+ s4 = 10–16
0.1–x x x + 0.1 s = 10–4
x(x  0.1)
1.75 × 10–5 =
0 .1  x 1
x = 1.75 × 10–5 16. pH = pKa  pKw  log C
2
1.75  10 5 1
% DOD = ×100 = 0.0175% = [4 + 14 + log 0.05]
0.1 2
pH = 8.34
6. (b) NaOH + HCl  NaCl + H2O
0.15 0.10 17. x(x + y) = 1.6 × 10–10
0.05 0
NaOH + CH3COONa  CH3cOONa + H2O 1.6  10 10
x= = 1.6 × 10–7
0.05 0.1 0.05 10  3
0 0.05 [Ag+] = 1.6 × 10–7
x=1
salt
7. pH = pKa + log 18. AgCl Ag+ + Cl–
acid
x x+y
= 5 – log1.75 + 0 CuCl Cu+ + Cl–
4.75 y x+y
y(x + y) = 10–6
8. T(H+) x(x + y) = 1.6 × 10–10
pH  (x + y)2 = 10–6 + 1.6 × 10–10
x + y = 10–3
9. x + H2O nx + OH–
0.1–x x x
x2
10–9 =
0.1  x

394 - Rajeev Gandhi Nagar Kota, Ph. No. 0744-2209671, 93141-87482, 93527-21564
IVRS No. 0744-2439051, 0744-2439052, 0744-2439053 www.motioniitjee.com, email-info@motioniitjee.com

You might also like